10 Landmark Decisions: Competence (Cloned) 10 Landmark Decisions: Competence (Cloned)

10.1 Dusky v. United States 10.1 Dusky v. United States

DUSKY v. UNITED STATES.

No. 504,

Misc.

Decided April 18, 1960.

James W. Benjamin for petitioner.

Solicitor General Rankin for the United States.

Per Curiam.

The motion for leave to proceed in forma pauperis and the petition for a writ of certiorari are granted. Upon consideration of the entire record we agree with the Solicitor General that “the record in this case does not sufficiently support the findings of competency to stand trial/' for to support those findings under 18 U. S. C. § 4244 the district judge “would need more information than this record presents.” We also agree with the suggestion of the Solicitor General that it is not enough for the district judge to find that “the defendant [is] oriented to time and place and [has] some recollection of events,” but that the “test must be whether he has sufficient present ability to consult with his lawyer with a reasonable degree of rational understanding — and whether he has a rational as well as factual understanding of the proceedings against him.”

*403In view of the doubts and ambiguities regarding the legal significance of the psychiatric testimony in this case and the resulting difficulties of retrospectively determining the petitioner’s competency as of more than a year ago, we reverse the judgment of the Court of Appeals affirming the judgment of conviction, and remand the case to the District Court for a new hearing to ascertain petitioner’s present competency to stand trial, and for a new trial if petitioner is found competent.

It is so ordered.

10.2 Rogers v. Okin 10.2 Rogers v. Okin

Rubie ROGERS, Willie Wadsworth, Donna Hunt, James Colleran, Harold Warner, Elizabeth Bybel, Able Bolden, for themselves and on behalf of all persons similarly situated, Plaintiffs, v. Robert OKIN, M.D., Commissioner of the Department of Mental Health of the Commonwealth of Massachusetts, Richard Kahn, M.D., William Malamud, M.D., David Seil, M.D., Michael Gill, M.D., Elliot Schildkrout, M.D., Sanford Pomerantz, M.D., Jean Tumquest, M.D., Allan Siegel, Eugene Cacciola, M.D., Brian Mazmanian, M.D., Michael Osborne, M.D., John Szlyk, M.D., William Kantar, M.D., John Goodman, M.D., Defendants.

CA 75-1610-T.

United States District Court, D. Massachusetts.

Oct. 29, 1979.

*1351Greater Boston Legal Services, Roxbury, Mass., Richard W. Cole, Robert Burdick, Mary Ashbury, and Shubow, Stahlin & Bergstresser, Inc., Clyde D. Bergstresser, Boston, Mass., for plaintiffs.

Marvin C. Guthrie, Needham, Mass., for guardian ad litem.

Powers & Hall, Douglas Danner, Spencer J. Dreischarf, Marcia Sneden, Boston, Mass., for defendants Drs. William Malamud, Brian Mazmanian, Eugene Cacciola, Jeffrey Goldbarg, John Goodman, Michael Osborne, Sanford Pomerantz and Jean Turnquest.

Francis X. Bellotti, Atty. Gen., Stephen Schultz, Leah S. Crothers, Asst. Attys. Gen., Boston, Mass., for defendants Drs. Michael Gill, William Kantar, John Szlyk, Elliot Schildkrout, David Seil, Richard Kahn, William Malamud, Allan Siegel, Brian Mazmanian, Sanford Pomperantz, Jean Turnquest and Robert Okin.

Finnerty & Finnerty, John F. Finnerty, Jr., Boston, Mass., for Dr. Michael Gill.

Ficksman & Conley, David M. Gould, Boston, Mass., for Drs. John Szlyk, Richard Kahn and Allan Siegel.

Martin, MaGunson, McCarthy & Kenney, Charles Reidy, III, Raymond J. Kenney, Jr., Boston, Mass., for Drs. Elliot Schlidkrout, John Goodman, Jeffrey Goldbarg, Eugene Cacciola and Michael Osborne.

TAURO, District Judge.

TABLE OF CONTENTS

I. Introduction 1352

II. Procedural Background 1353

III. The Parties 1354

A. Plaintiffs 1354

B. Defendants 1354

IV. Boston State Hospital Facilities and Personnel 1355

A. The Austin Unit 1356

B. The May Unit 1357

V. Procedures for Commitment to the Boston State Hospital 1358

A. Conditional Voluntary Admission 1358

B. Involuntary Temporary Hospitalization for 10 Days 1358

C. Involuntary Prolonged Civil Commitment 1358

D. Commitment of Alleged Alcoholics 1358

E. Admission and Retention of the Mentally Retarded 1358

VI. Department of Mental Health Regulations Concerning Treatment at Boston State Hospital 1359

VII. Anti-Psychotic Drugs 1359

VIII. The Injunction Claim against Medication Practices 1360

A. The Competency of Mental Patients to Refuse Treatment 1361

B. Guardianship 1362

C. The Right to Refuse Treatment in an Emergency 1364

D. The Right to Refuse Treatment in a Non-Emergency 1365

1. The Involuntary Patient’s Right to Refuse Treatment 1365

a. The Involuntary Patient’s Right to Privacy 1365

b. The Involuntary Patient’s First Amendment Rights 1366

2. The Voluntary Patient’s Right to Refuse Treatment 1367

E. The Commonwealth’s Interests 1368

*1352IX. Seclusion 1371

A. Seclusion Statutes and Regulations 1371

B. Seclusion Facilities at the Austin and May Units 1372

C. Seclusion Practices at the Austin and May Units 1373

1. The Austin Unit 1373

2. The May Unit 1373

D. Discussion 1374

X. Named Plaintiffs’ Claims for Damages 1375

A. Findings of Fact Related to Damages Claims 1375

1. May Unit Plaintiffs 1375

a. Medication 1375

b. Seclusion 1376

2. Austin Unit Plaintiffs 1377

a. Medication 1378

b. Seclusion 1378

B. Legal Conclusions Related to Damages Claims 1380

1. Plaintiffs’ Federal Claims 1380

2. Plaintiffs’ State Claims 1383

a. Assault and Battery, False Imprisonment 1383

b. Malpractice 1384-

1. Medication Negligence Claims 1386

2. Seclusion Negligence Claims 1388

OPINION

I. INTRODUCTION

This class action involves a multi-faceted attack against certain medication and seclusion policies allegedly followed at the May and Austin Units of the Boston State Hospital (Hospital), a state institution for the mentally ill. The named plaintiffs, all either voluntary or involuntary patients at one time or another at these facilities, seek injunctive relief for the class,1 and award of money damages for themselves.

Plaintiffs’ basic grievance is that the defendants, all of whom have served on the Hospital staff, maintained policies of forced medication and involuntary seclusion in non-emergency circumstances. Plaintiffs allege that these policies infringed on the constitutional rights of Hospital patients. In addition, they allege that such policies violated standards of acceptable medical care.

With respect to the challenged medication practices, plaintiffs theorize that, although they have a right to receive treatment when confined at a state mental institution, they, nonetheless, have a constitutional right to refuse such treatment. Plaintiffs acknowledge, however, that their asserted right to refuse treatment is not absolute, and must yield to the Hospital’s right to impose treatment in order to protect their safety or that of other patients and Hospital staff. Absent such emergency circumstances, plaintiffs maintain they are competent to decide whether or not to receive certain treatment, and that their decisions must be respected by Hospital staff.

As for the seclusion issue, plaintiffs maintain that state law permitted defendants to restrain patients in seclusion rooms only when there was a substantial threat of physical harm to patients or staff.2 Plaintiffs allege that, notwithstanding such stat*1353utory proscription, defendants routinely employed seclusion as a treatment modality, and not merely as an emergency restraint.

The defendants have primary and fall back positions with respect to plaintiffs’ allegations and claims. Their fundamental defense is that patients committed to a state mental institution, whether voluntary or involuntary, are incompetent to make treatment choices. Defendants assert that mental patients are committed to mental hospitals for treatment, and that the state has a parens patriae obligation and right to provide that treatment, even in the face of opposition by the patient. In short, defendants argue that committed mental patients 3 have no constitutional right to refuse treatment in either an emergency or non-emergency situation. Defendants concede, however, that any treatment provided must be consistent with reasonably accepted standards of medical practice.

In addition to their legal contention, defendants offer a factual defense to plaintiffs’ medication claims. They maintain that none of the named plaintiffs was forcibly medicated in a non-emergency. Moreover, defendants assert that no patient at the Hospital was forcibly medicated unless there was at least a “psychiatric emergency,” a term they define as the foreseeable deterioration of the patient absent medication.

Concerning the seclusion issue, defendants concede that M.G.L.A. ch. 123, § 21 is the controlling standard. They maintain, however, that no patient was secluded in violation of that standard.

The examination of these medication and seclusion issues involved 72 trial days, more than 8,000 pages of transcript and over 2,300 pages of post-trial briefs. The findings and conclusions of this court concerning these issues are set forth below.

II. PROCEDURAL BACKGROUND

This action was commenced on April 27, 1975, when several patients at the May and Austin Units of Boston State Hospital filed a civil rights action under 42 U.S.C. § 1983 seeking to enjoin certain seclusion and medication practices at the Hospital and to recover compensatory and punitive damages from those responsible for such practices.

On April 30, 1975, this court issued a temporary restraining order prohibiting non-emergency seclusion and medication of voluntary or involuntary patients without their informed consent, or that of a guardian in the case of an incompetent. On May 8, 1975, the parties agreed to an extension of the temporary restraining order until a hearing on preliminary or permanent relief was concluded. Such a hearing commenced in the fall of 1975 and continued for a period of six trial days. Further hearing was then suspended while the court and the parties attempted to settle the complex issues involved. Those efforts continued until June of 1976 when, because of seemingly irreconcilable differences, a merged trial on preliminary and permanent injunctive relief was scheduled for June 21, 1976.

Subsequent to that scheduling, defendants filed motions for summary judgment on all damages claims and on the injunctive claims relating to the use of seclusion at the Hospital. The June 21, 1976 date was utilized for hearing on these motions rather than trial on the merits. The motions were denied on March 25, 1977.

On March 25, 1977, this court also denied defendants’ motion to dissolve the temporary order restraining forced medication. That decision was appealed and arguments were held on September 9,1977 by the First Circuit Court of Appeals. An order affirming this court was issued on December 8, 1977.

All action in the case had been stayed pending appeal. Trial on the merits commenced in December of 1977 and concluded on January 31, 1979. Seventy-two trial days were dedicated to the testimony of more than 50 witnesses, most of whom were *1354psychiatrists, psychologists or other professionals. During the next several months, the parties prepared extensive post-trial briefs and proposed findings of fact. The case was taken under advisement on August 15, 1979, following final oral argument.

III. THE PARTIES

A. Plaintiffs

ABLE BOLDEN, age 46, was first hospitalized at Massachusetts Mental Health Center in 1953. During the next twenty years, he was admitted to various state hospitals, primarily because of episodic violent behavior. In December of 1974, he was involuntarily committed to the May Unit for 10 days. On January 3, 1975, Bolden was again admitted to the May Unit, this time for a 20 day observation which was extended to February 12, 1975. On February 22, 1975, he was involuntarily committed on a temporary 10 day admission,4 and on March 25,1975, he was again involuntarily committed, this time for six months.

BETTY BYBEL, age 38, was first admitted to the Austin Unit for observation by a court order on January 3, 1973. She had two prior hospitalizations for the treatment of mental illness, first in New Jersey (1967) and again in Massachusetts (1972). Between January 3, 1973 and April 25, 1975, she was admitted to the Austin Unit on approximately 28 occasions, sometimes voluntarily and sometimes involuntarily.

JAMES COLLERAN, age 21, was first admitted to the Austin Unit for six months. From August 11, 1974 to December 10, 1974, he was held at the Austin Unit pending a court commitment hearing. That hearing resulted in an involuntary commitment through June 9, 1975. He is one of eleven children. Both of his parents, now dead, suffered from mental illness. For the past ten years, he has had many encounters with the juvenile court system. Prior to his Austin admission, he was involved in episodes of violence.

DONNA HUNT, age 20, was first admitted to the Austin Unit in January 1974 on an involuntary ten day commitment. Her mother signed a conditional voluntary commitment contract on her behalf in January of 1974. After becoming 16, Donna agreed to a voluntary commitment. She remained in that status from May 1974 until August 1975, when she was involuntarily committed to the Austin Unit. When she was three years old she contracted measles encephalitis, which apparently resulted in organic brain damage. Her record demonstrates many instances of violent behavior.

RUBIE ROGERS, a woman in her late thirties, has had a number of May Unit admissions and discharges since 1965. From January 1971 to April 1975, she was a voluntary patient. She has had a history of thought disorder, hallucinations and delusions, with occasional episodes of self-destructive and violent behavior.

WILLIE WADSWORTH, age 29, was transferred to the May Unit from Bridge-water State Hospital on April 17, 1974, and was involuntarily committed until April 16, 1975. A large and powerfully built man, he was first admitted to a mental hospital at age 17.

HAROLD WARNER, age 52, spent seventeen years at Bridgewater State Hospital after conviction of assault and battery of a twelve year old girl. In August of 1974, he was released by order of the Superior Court and admitted to the May Unit as an involuntary patient.

B. Defendants

EUGENE CACCIOLA received his M.D. in 1974 from Tufts University. From January to July, 1975, he was a resident on ward 4 of the May Unit. Claims have been brought against him by Rubie Rogers, Able Bolden, and Harold Warner.

MICHAEL GILL received his M.D. from the Royal College of Surgeons in Dublin, Ireland. He took his residency at Boston State Hospital from 1959 to 1962 and became board certified in psychiatry in 1964. *1355He founded the Austin Unit in 1966 and held the position of director until 1977. Dr. Gill is being sued by Donna Hunt, James Colleran, and Elizabeth Bybel.

JOHN GOODMAN received his M.D. from Boston University in 1974. From January to June, 1975, he was a resident on ward 3 of the May Unit. Claims against him have been brought by Willie Wads-worth and Able Bolden.

RICHARD J. KAHN received his M.D. from Harvard University in 1955. He spent two years as a psychiatric resident at Boston State Hospital. From January 1970 through August of 1974, he was inpatient director of the May Unit. He is board certified in psychiatry. Claims have been brought against him by Rubie Rogers and Harold Warner.

WILLIAM G. KANTAR received his M.D. in 1962 from Tufts University. From 1969 to January of 1977, he was a member of the staff of the Austin Unit, serving as senior and clinical director of psychiatry. He has been board certified in psychiatry since 1963. Claims have been brought against him by Donna Hunt, James Colleran, and Elizabeth Bybel.

WILLIAM MALAMUD received his M.D. from Boston University in 1954. From 1955 to 1956, he was a psychiatric resident at Boston State Hospital. On September 1, 1974, Dr. Malamud became assistant superintendent for clinical affairs at the Solomon Carter Fuller Mental Health Center. From September 1, 1974, when Dr. Kahn left the May Unit, until March 24, 1975, when Dr. Seil became the inpatient director, Dr. Malamud was the acting interim director of the inpatient service at the May Unit. Dr. Malamud was board certified in psychiatry in 1964, and certified to practice psychoanalysis in 1969. Claims have been brought against him by Rubie Rogers, Willie Wads-worth, Able Bolden and Harold Warner.

BRIAN MAZMANIAN received his M.D. from St. Louis University in 1974. He was a resident on ward 4 at the May Unit from January, 1975 to July 30,1975. A claim has been brought against him by Able Bolden.

MICHAEL OSBORNE received his M.D. from Michigan State University in 1972. He served as resident at the May Unit from July 1, 1974 to June 30, 1975. Claims have been brought against him by Rubie Rogers and Able Bolden.

SANFORD POMERANTZ received his M.D. from Boston University in 1973. From July 1, 1974 to January 6, 1975, he was a resident at the May Unit. A claim has been brought against him by Willie Wadsworth.

ELLIOT SCHILDKROUT received his M.D. from New York University in 1973. From January to July 1975, he served as a resident on ward A-3/A — 4 of the Austin Unit. Claims have been brought against him by Donna Hunt, James Colleran and Elizabeth Bybel.

DAVID SEIL received his M.D. from Yale University in 1962. From January 1974 until March 1975 he was director of the Evaluation Service at the May Unit. He served for the next three years as director of inpatient services at the May Unit. A claim has been brought against him by Harold Warner.

ALLAN SIEGEL received a doctorate in counseling from Boston University. Dr. Siegel served as supervisor of ward 3 in the May Unit from June 1974 to June 1976. He is a licensed psychologist. A claim has been brought against him by Willie Wadsworth.

JOHN SZLYK received his M.D. from Tufts University. From July to December 1974, he served as a resident at the Austin Unit on ward A-4. A claim has been brought against him by Donna Hunt.

JEAN TURNQUEST received her M.D. from Aberdeen University in 1971. From July 1, 1974 to January 5, 1975, she was a resident on ward 3 of the May Unit. A claim against her has been brought by Willie Wadsworth.

IV. BOSTON STATE HOSPITAL FACILITIES AND PERSONNEL

Boston State Hospital is a multi-unit campus type facility located in the Dorchester section of Boston. From 1973 to 1975, *1356Austin and May were two of these units. By the end of 1975, Austin was affiliated solely with Tufts Bay Cove Mental Health Center, while May became part of the Solomon Carter Fuller Mental Health Center.

The May Unit was a teaching facility utilized by the Boston University Medical School. Austin had a comparable affiliation with Tufts. Both served as state hospitals for mentally ill adults, age 16 and over. The population of each is determined by the residence of the patient. The Commonwealth is divided into mental health regions which, in turn, are subdivided into “catchment areas.” Absent special circumstances, patients may receive treatment only in the catchment area that includes their residence. The catchment area served by Austin consisted of South Boston and parts of Chinatown and North Dorchester. The catchment area for the May Unit consisted of parts of Back Bay, the South End, most of Roxbury, and parts of Dorchester.

A. The Austin Unit

Austin was built in 1918-20 as a maximum security ward for 160 female patients. It is a drab, gloomy, poorly lighted structure in a state of chronic disrepair. Poor lighting restricted evening recreational opportunities. Plumbing problems were constant, often resulting in basement flooding and the odor of sewage through the building. As of January 1975, wards A-3 and A-4 were on the second floor while A-6 was on the third floor. The ward staff of A-3 and A-4 was divided into three teams to handle the approximately 50 patient case load. The Austin Unit was moved to a new location in 1977. Its name was changed to the Johnson Unit.

Between January 1973 and April 1975, Austin’s daily census averaged about 75 inpatients. Of the approximately 500 admissions per year, about 60% were re-admissions. As of 1974, the average length of hospital stay was 14 days, down from an average of 17 days in 1972. Between January 1974 and January 1975, there were approximately 35-45 patients per day on ward A-4, about half of whom were females. Most of these were over 25 years old. At any given time, four or five might be under 20 years of age. There was rarely a patient under 16. Approximately 20 to 25% of the patient load were potentially violent, with about 3 to 5% actually engaging in violent behavior periodically.

From January 1973 to April 1975, the number of direct care staff at Austin averaged between 60-65. Twenty-three full-time nurses were on duty on three eight-hour shifts. Two physicians were assigned to each ward, as well as a staff psychiatrist, resident psychiatrist, at least two registered nurses, one social worker, a psychologist, a rehabilitation counselor, and mental health workers. A nurse would be present 90-95% of the daytime and a doctor was always available seven days a week. There was an average of four staff people for each ward during the evening shift and about three during the night shift. A “team system” of patient care was utilized. Each team was headed by a resident and included members of the various professional disciplines working within the hospital.

The defendant Dr. Gill was the Austin unit chief from January 1973 through January 1975. As such, he had overall responsibility and authority. In addition, he had primary psychiatric supervisory responsibility for ward A-6.

The defendant Dr. Kantar became responsible for the merged ward A-3 and A-4 and Dr. David Curtis took over responsibility for ward A-6 as of January 1975. Thereafter, Dr. Gill no longer had responsibility for any particular ward, but made building rounds three days a week. He did not attend daily ward rounds.

Dr. Kantar, the staff psychiatrist, supervised the psychiatric residents. These residents served for terms of six months. They included Dr. Press (not a defendant) from January 1974 until June 1974; the defendant Dr. Szlyk from July 1, 1974 until the end of December 1974; and the defendant Dr. Schildkrout from January 1973 until June 1975.

*1357In the fall of 1974, Layne Erban became the director of nursing of the Austin Unit, and had administrative responsibility for the entire Austin nursing and attendants staff. Her responsibilities included scheduling, coverage and policy. In addition, she participated in the activities of the ward A-d until its merger with A-3. Although Erban did not have line authority over residents, she was one of their prime sources of information with respect to practices and policies at Austin. If a resident should fail to follow those practices and policies, she would inform Dr. Kantar or Dr. Gill.

Each ward had a head nurse to whom responsibilities were delegated by Erban. Essentially, the head nurse was the ward administrator, particularly with respect to scheduling patient treatment and general implementation of policy. Dr. Gill had line authority over Layne Erban and each of the ward administrators after the fall of 1974. Prior to this time he did not have the power to hire or fire nurses.

B. The May Unit

The May Unit was in a three storied U-shaped building. Offices and meeting rooms were on the first floor. The wards were on the second and third floors, and had an inpatient capacity of about 90. Wards 3 and 4, prime subjects of this litigation, were on the second floor. Overall, the building was in chronic disrepair. The heating system was particularly unreliable.

The May staffing pattern was headed by a clinical director. Under him was the senior ward psychiatrist who, in turn, supervised the resident psychiatrists.

The nursing staff reported to the director through the nursing director. Subordinate to her were the ward head nurses who, in turn, supervised the ward LPNs. At the bottom of the totem pole were the mental health workers.

The director had overall responsibility for clinical and administrative management of the staff and plant. This responsibility included consultations with and supervision over the psychiatric and nursing staff, as well as the creation and implementation of treatment policies.

From 1970 through most of 1974, Dr. Kahn was director of the May Unit. In September 1974, the defendant Dr. Malamud became acting director until the defendant Dr. Seil assumed the post in March 1978.

Dr. Siegel, a psychologist, was in charge of ward 3. He had primary responsibility for the clinical activities on the ward. His position included the supervision of resident psychiatrists and consultation with staff and patients with respect to development of policies. He would also preside at weekly ward meetings. Although his primary responsibility involved ward 3, Dr. Siegel also served as a consultant to Drs. Mazmanian and Cacciola, the residents on ward 4, as well as the resident on ward 6, Dr. Holstein. Dr. Siegel supervised Drs. Pomerantz and Tumquest when they were ward 3 residents, as well as their replacements, Drs. Goodman and Goldberg. He also supervised two ward 4 psychologists. Dr. Siegel did not have supervisory responsibility for the nursing staff.

Each ward 3 resident was responsible for the medical and psychiatric care of approximately 10-12 patients. On ward 4, three teams functioned, two under direction of Drs. Cacciola and Mazmanian. The third was supervised by a social worker. The per resident patient load on ward 4 varied from a low of eight to a high of 15 during the first half of 1975. The May Unit daily census between January 1972 and September 1974 varied between 75 and 100 patients. From January to June 1975, the census averaged about 120 patients, approximately 25 of whom were on ward 3.

The typical day shift averaged three to five in staff, in addition to two or three doctors. There were slightly fewer staff on the evening and night shifts, although doctors were “on call” on a rotating basis. The average patient census was 21. Staff had a number of additional responsibilities that required their presence outside the ward.

As of late 1974, May was without a full time clinical director, and suffered a short*1358age of full time senior staff direction in wards 3, 4, and 6. There were three head nurses for the entire Unit. The ward 3 day shift had to make do with a total of six to eight licensed practical nurses and attendants. On the second and third shifts, there were about four to six LPNs and attendants.

Each ward had two or three treatment teams. Typically, they would include resident psychiatrists, psychologists, nursing staff, social workers and the patients. The teams met twice a week to discuss details of particular patient treatment plans. Most treatment planning took place during the day shift. There were ward intershift meetings primarily involving the 7-to-3 and 3-to-ll shifts. The ll-to-7 and 7-to-3 shifts also met on the average of once a month.

A number of activity programs were available for patients at both Austin and May. Those available at Austin were art therapy, home care workshop, drama, exercise, plants, pottery, daily living, newspaper and sports. Among the activities available at May were ward meetings, team meetings, individual meetings between a patient and his or her administrator, occupational therapy, daily living, physical therapy, rehabilitation therapy, music, games and alcoholics meetings.

V. PROCEDURES FOR COMMITMENT TO THE BOSTON STATE HOSPITAL

Standards for commitment to state-run mental institutions in Massachusetts are defined by statute. Most admissions fall under one of the following categories:

A. Conditional Voluntary Admission

M.G.L.A. ch. 123, §§ 10, 11

Patients needing treatment for mental illness are encouraged to volunteer for commitment. Application may be made by a parent or guardian, although their approval is unnecessary if the applicant is 16 years or older. The hospital may be inspected by the prospective voluntary applicant prior to admission. The voluntary patient may be discharged upon request, after giving three days notice to institution authorities.

B. Involuntary Temporary Hospitalization for 10 Days

M.G.L.A. ch. 123, § 12

An involuntary 10 day commitment of a person may be obtained upon a physician’s statement that, absent hospitalization, a likelihood of serious harm exists due to mental illness. After admission, the patient is examined for 10 days to determine whether failure to hospitalize the patient further would create a likelihood of serious harm due to mental illness. If such a likelihood exists, the hospital may petition the court for civil commitment.

C. Involuntary Prolonged Civil Commitment

M.G.L.A. ch. 123, §§ 7, 8

If the hospital administrator determines that continued hospitalization of a patient is necessary because a likelihood exists of serious harm due to mental illness, he may petition the court for the patient’s commitment. The court will notify the patient and the patient’s nearest relative or guardian who will have the opportunity to request a hearing. The court may order the patient committed for a period of up to one year.

D. Commitment of Alleged Alcoholics

M.G.L.A. ch. 123, § 35

A police officer or a physician may petition any district court for commitment of a person deemed to be an alcoholic. If the court finds there is a likelihood of serious harm because of alcoholism, that person may be ordered committed for up to 15 days.

E. Admission and Retention of the Mentally Retarded

All admissions of the mentally retarded are voluntary unless the hospital determines that the retarded person is also mentally ill and that release would create a likelihood of serious harm. In such a case, the patient would be required to submit a *1359three day notice of intention to leave and the hospital administrator could petition the court for permission to retain custody.

VI. DEPARTMENT OF MENTAL HEALTH REGULATIONS CONCERNING TREATMENT AT BOSTON STATE HOSPITAL

Commitment to Boston State, even on an involuntary basis, is not an adjudication of incompetence. Department of Mental Health (D.M.H.) Regulation § 221.02 states:

Civil Rights. No person shall be deprived of the right to manage his affairs, to contract, to hold professional, occupational or vehicle operator’s licenses, to make a will, to marry, to hold or convey property, or to vote in local, state, or federal elections solely by reason of his admission or commitment to a facility except where there has been an adjudication that such person is incompetent, or when a conservator or guardian has been appointed for such person. In the event of conservator-ship, a patient’s civil rights may be limited only to the extent of the conservator’s adjudicated responsibility.

The substance of this section has been codified in M.G.L.A. ch. 123, § 25.

A committed patient has the right to receive “treatment suited to his needs which shall be administered skillfully, safely, and humanely with full respect to his dignity and personal integrity.” D.M.H. Reg. § 221.03. D.M.H. regulations also provide that a committed person

shall receive treatment and rehabilitation in accordance with accepted therapeutic practice, including oral, subcutaneous and intramuscular medication when appropriate and when ordered by a physician. However, electroconvulsive treatment and lobotomy shall require separate consent by the patient pursuant to M.G.L.A. ch. 123, § 23.

D.M.H. Reg. § 220.02. Another D.M.H. regulation requires that

[e]ach facility under the supervision and control of the Department or licensed by the Department shall post a copy of the rights articulated in this regulation in the admission room of the facility in each residential unit or any other appropriate places in the facility.

D.M.H. Reg.M.H.16-7. In accordance with that requirement, a statement of patients’ civil rights was posted on the wards of the May and Austin Units, and was included in a packet of information presented patients on admission. The poster stated:

Your Rights
You Have The Right To Be Treated With Dignity And Respect
You Have the Right to Privacy
TREATMENT RIGHTS:
You have the right to:
—be told in detail what is wrong with you, what alternative treatments are available, and to choose from these alternatives.
—be informed of the risks and possible side effects of treatment, and to refuse treatment at any point.
Restraints and Seclusion: You may be restrained only in case of emergency. Such restraints or seclusion must be justified in your record, and must be reviewed by the superintendent within 8 hours.

The admission packet stated:

PROCEDURES FOLLOWING ADMISSION:
A course of treatment may include counselling, medications, electroconvulsive treatment, and treatment of any medical condition that the patient may be found to have.
No medical or surgical treatments including electroconvulsive treatment are given without the consent of the patient.

VII. ANTI-PSYCHOTIC DRUGS

The plaintiffs’ principal objection to forced medication is the potential for anti-psychotic drugs to cause certain adverse side effects. Because of that adverse potential, plaintiffs maintain that the decision whether to reject or accept medication in a non-emergency situation should rest with the patient.

*1360Anti-psychotic drugs are chemical agents used to manage and treat serious mental illness. They are also referred to as neuroleptic drugs and psychotropic drugs. The drugs that the plaintiffs received included Thorazine, Mellaril, Prolixin and Haldol. In general, the drugs influence chemical transmissions to the brain, affecting both activatory and inhibitory functions. Because the drugs’ purpose is to reduce the level of psychotic thinking, it is virtually undisputed that they are mind-altering.

Foremost among the possible side effects of anti-psychotic drugs is tardive dyskinesia. Tardive dyskinesia is a neurological side effect which may appear after prolonged use of anti-psychotic drug treatment. The disease is the outcome of a complex patient-drug interaction which is not currently well understood.5 The overt symptoms of tardive dyskinesia include certain involuntary motor movements, particularly of the face, lips, and tongue. Tardive dyskinesia can also cause the involuntary movement of fingers, hands, legs and the pelvic area. In its most progressive state, the disease can interfere with swallowing and can affect all motor activity. While in mild cases the disease can simply be a source of embarrassment, it can be physically and psychologically disabling.6 Until very recently, tardive dyskinesia was considered irreversible. Some studies now suggest that in certain cases it can be effectively treated.7

Recent studies also suggest that tardive dyskinesia is more widespread in mental patients than previously considered. Two studies now place the prevalence of tardive dyskinesia among chronically hospitalized schizophrenics at 50% and 56%. With respect to outpatients, one survey has reported a prevalence rate of 41%.8 Although this court is unpersuaded that any of the named plaintiffs suffered from tardive dyskinesia, see Sec. XI (A) infra, several of the defendants have admitted that other patients at Boston State suffered from the disease.

There are also a variety of neurological side effects of anti-psychotic drugs, known as extrapyramidal effects. These include akathisia (motor restlessness — the inability to sit still), akanesia (physical immobility and lack of spontaneity), dystonia (spasmodic muscle reaction frequently characterized by a twisting of the neck) and pseudoparkinsonian syndrome (mask-like face, rigidity of the hand). These conditions are not considered to be irreversible.

VIII. THE INJUNCTION CLAIM AGAINST MEDICATION PRACTICES

Plaintiffs allege that the defendants have impermissibly followed a policy of forcibly medicating committed mental patients, and that such policy has denied them their constitutionally protected right to refuse treatment.9 Although plaintiffs urge this court to recognize a right to refuse treatment, they do not maintain that such a right is absolute. They acknowledge that in emergencies it must yield to the state’s interest in medicating.

Defendants proffer a three-pronged defense to plaintiffs’ allegations. First, they maintain that a committed mental patient is per se incompetent to decide whether or not to receive treatment. Second, they deny that any patient was forcibly medicated except in circumstances amounting to at least a psychiatric emergency. Third, they *1361assert that committed mental patients, whether voluntary or involuntary, have no constitutional right to refuse treatment in any situation — emergency or non-emergeney.

The respective positions of the parties set up the fundamental issue as to when, if ever, an institutionalized mental patient may be forcibly medicated10 in non-emergencies. Prior to discussing the legal question of whether there exists a fundamental right to refuse psychotropic medication, it is necessary to consider certain threshold issues.

A. The Competency of Mental Patients to Refuse Treatment

A pivotal issue dividing the parties in this case is the competency of mental patients to decide rationally whether or not to receive treatment. At final argument, defendants took the flat position that, once admitted to a mental institution, a patient is deemed incompetent to decide whether or what to accept by way of treatment in either an emergency or non-emergency situation. And in their Trial Brief, defendants asserted that

[t]he Commonwealth is not seeking to forcibly or involuntarily medicate patients competent to decide for themselves. (Defendants’ Trial Brief p. 27) (emphasis supplied).

Hence, defendants argue that plaintiffs as incompetents cannot assert any constitutional right to refuse treatment. Basically, defendants theorize that,

once an individual becomes incompetent, the state must act as parens patriae. As such, it has the duty and the right to care for the ‘best interest’ of the incompetent, even if that occasionally means overriding a decision made by the individual while incompetent.

In re Boyd, 403 A.2d 744, 748 n. 8 (D.C.App. 1979) (citations omitted),

Plaintiffs disagree. They argue that, as a matter of state law, mental patients are presumed competent to manage their affairs,11 and that such presumption must be deemed to include competence to make treatment decisions.

The weight of evidence persuades this court that, although committed mental patients do suffer at least some impairment of their relationship to reality, most are able to appreciate the benefits, risks, and discomfort that may reasonably be expected from receiving psychotropic medication. This is particularly true for patients who have experienced such medication and, therefore, have some basis for, assessing comparative advantages and disadvantages. Indeed, a fundamental concept for treating the mentally ill is the establishment of a therapeutic alliance between psychiatrist and patient. Implicit in such an alliance is an understanding and acceptance by the patient of a prescribed treatment program.

Moreover, defendants’ position that commitment per se. demonstrates the incompetence of a mental patient to decide treatment questions is at odds with M.G. L.A. ch. 123, § 25 and D.M.H. Reg. § 221.02. These provisions state unequivocally that, although committed, a mental patient is nonetheless presumed competent to manage his affairs, dispose of property, carry on a licensed profession, and even to vote. That presumption of competency prevails unless and until there has been an adjudication of incompetency by a court, following notice and hearing. D.M.H. Reg. §§ 221.06, 222.-05.12

To be sure, these regulatory provisions do not expressly grant mental patients a right to refuse treatment, except with *1362respect to electrical shock and lobotomy. But, M.G.L.A. ch. 123, § 25 and related D.M.H. regulations do recognize in absolute terms the competence of committed persons to manage their affairs and participate in a variety of challenging activities. That recognition tilts the scales in favor of presuming, as well, the competence of a committed mental patient to make treatment decisions, absent an adjudication to the contrary.13

B. Guardianship

The D.M.H. regulations provide that a patient’s competence may be called into question. Indeed, they impose an obligation upon the institution to review periodically a patient’s competence to “manage rationally the ordinary affairs of life,” D.M.H. Reg. § 222.05, and to seek the court appointment of a guardian if the competence of a patient is in doubt.

Assuming an adjudication of incompetence is made, a guardian would be appointed to manage the patient’s affairs. Plaintiffs contend that the guardian could make medication decisions on behalf of the incompetent patient in nonemergencies, thereby serving the Commonwealth’s parens patriae interest in treating the patient. Given an emergency, plaintiffs concede that the Commonwealth would have a right to forcibly medicate any committed patient, competent or otherwise. Thus, plaintiffs contend that the Commonwealth’s interest in preserving order and safety at the Hospital is adequately served as well.

The defendants have both legal and factual objections to the reliance on guardians in the treatment process. First, defendants contend that, even assuming committed patients have a right to refuse treatment, such right would be “personal” and not within the representative powers of even a judicially appointed guardian. Second, they contend that the guardianship scheme has proved impracticable.

In support of their legal theory, defendants cite the following passage from Eisenstadt v. Baird, 405 U.S. 438, 453, 92 S.Ct. 1029, 1038, 31 L.Ed.2d 349 (1972):

If the right of privacy means anything, it is the right of the individual, married or single, to be free from unwarranted governmental intrusion into matters so fundamentally affecting a person as the decision whether to bear or beget a child. (Emphasis in original).

The defendants, however, miss the import of the court’s admonition. The key to the Eisenstadt quote is that the right of privacy means the right of individuals to be free from unwarranted governmental intrusion in fundamental personal matters. The fact that a court appointed guardian may have to act to protect the patient in no way abrogates his basic right to be free from unwarranted government intrusion. To decide otherwise would make defenseless not only judicially declared incompetents, but small children whose interests traditionally have been represented and protected by parents and guardians. Defendants’ theory would essentially require such persons to fend for themselves.14

In making medication decisions, a guardian would not act as a third person, but would merely stand and act in the place of the patient. The patient’s right is not to get a guardian. It is to be free from unwarranted government intrusion. The guardian is merely a means for protecting that freedom.

Distinguishable as well is Parham v. J.L. and J.R., - U.S. -, 99 S.Ct. 2493, 61 L.Ed.2d 101 (1979), where the Supreme Court sustained the validity of Georgia’s commitment procedures for minors. While *1363it is true that the Court rejected the argument that a neutral person must decide whether or not a child should be committed to a mental hospital, it in no way suggested that the commitment decision could be made solely by the physician. There, the decision to commit was based on the collective opinion of physician and parents. Here, the appointment of a guardian would fulfill the parent’s responsibility recognized in Parham.

Most of the defendants’ factual contentions concerning the impracticality of utilizing guardians in the treatment process relate to the manner in which guardians are appointed. These contentions actually amount to little more than complaints concerning the effectiveness of a procedural framework designed and implemented by the Commonwealth.

The presumption that an involuntary mental patient is competent to handle his affairs is not a matter of judicial decree. Rather, it is a statutory presumption created by the Massachusetts legislature, as is the procedure for challenging that presumption in the courts. If that statutory scheme is burdensome, redress and relief should be sought from the legislature.

Similarly, defendants’ complaint that guardianship applications are unduly delayed in the state court is a matter within the Supreme Judicial Court’s general powers of superintendence and should be brought to that court’s attention.15 The same may be said for any complaint as to the ability or integrity of the potential guardian appointment pool. Moreover, defendants’ assertion that court appointed guardians are “unsuited for making decisions regarding whether medication should be imposed on a patient” (Defendants’ Trial Brief p. 43) is presumptuous and inconsistent with the increasingly accepted doctrine of informed consent which recognizes that lay persons are presumed to have the capacity to make treatment decisions.16

Also to be borne in mind is that juries and judges, traditionally non-medical persons, are routinely called upon to pass judgment in medical malpractice cases. Indeed, the Commonwealth’s statutory procedure for screening such cases provides for an examining panel of three persons, two of whom may be non-physicians.17

Without merit, as well, is defendants’ contention that hospital staff would be required to “run back and forth to the courts every time a patient’s condition improves or deteriorates.” (Defendants’ Trial Brief p. 43). Once a guardian is appointed, no further court appearances would be required. Of course, it might be necessary for the hospital staff to maintain a line of communication with the guardian. To do so, however, would impose no greater burden than that assumed anyway by most responsible doctors who routinely consult with parents and family members, in non-emergencies, with respect to a patient’s condition and course of treatment.

This court is similarly unpersuaded by defendants’ pleas as to the anti-therapeutic effects of appointing a guardian. Certainly, labeling a person as incompetent has a profound effect on his life, and may well have some impact on that person’s self-respect. But, we are not considering that phenomenon in isolation. We are dealing with the relative well-being of a patient who has already suffered the trauma of being confined to an institution and is now facing the experience of being forcibly disrobed and then injected with psychotropic medication against his will. Balancing all the circumstances, it is difficult to see how the incompetent mental inpatient would be damaged by having a guardian appointed to ensure against any unwarranted intrusion into his life.

*1364This court concludes, • therefore, that committed mental patients are presumed competent to make decisions with respect to their treatment in non-emergencies. Given an adjudication of incompetence, a guardian may exercise for and on behalf of a committed mental patient any rights he may have to make treatment decisions in a non-emergency.18

C. The Right to Refuse Treatment in an Emergency

As was noted at the outset of this opinion, the parties agree that forced medication is permissible in an emergency situation. They disagree, however, as to what circumstances amount to an emergency situation justifying such treatment.

Plaintiffs urge this court to define an emergency justifying forced medication as one in which there exists a substantial likelihood of personal injury to the subject patient, other patients or staff members. Plaintiffs recognize that medicine is an inexact science in which prognoses cannot be made with mathematical certainty. But, plaintiffs say that the threat of physical harm must, at the least, be more likely than not before there may be forced medication. Such a standard amounts to an expression of “probability,” long a common bench mark for the admissibility of expert testimony concerning medical issues. Basically, the plaintiffs urge that emergency standards that control the imposition of seclusion19 be applied as well to the issue of forced medication.

Defendants disagree, arguing that seclusion is a restraint while medication, forced or voluntary, is treatment. It would be inappropriate, defendants maintain, to impose on a treatment situation the strict standards for applying restraints. Instead, the defendants urge this court to recognize a broader definition of emergency — a so called “psychiatric emergency.” Under defendants’ theory, a psychiatric emergency justifying forced medication would exist given any of the following situations: 1) suicidal behavior, whether seriously meant or a gesture, 2) assaultiveness, 3) property destruction, 4) extreme anxiety and panic, 5) bizarre behavior, 6) acute or chronic emotional disturbance having the potential to seriously interfere with the patient’s ability to function on a daily basis, 7) the necessity for immediate medical response in order to prevent or decrease the likelihood of further severe suffering or the rapid worsening of the patient’s clinical state.

Although the defendants maintain that none of the named plaintiffs was forcibly medicated except when there was a serious threat of personal injury (the seclusion standard), they do admit that other class plaintiffs at Boston State have been forcibly medicated in circumstances that would not have justified the use of seclusion. (Defendants’ Trial Brief pp. 14 — 15). But, defendants insist that any forced medication occurred only in circumstances that would fall within their proffered definition of a psychiatric emergency. Basically, the defendants say that no patient at Boston State was ever forcibly medicated except when there was a sincere belief by defendants that such treatment was necessary to the patient’s recovery, or to provide urgently needed protection to that patient or others.

The court recognizes that varying degrees of crisis may typify the average day on a ward of any mental institution. Patient behavior can be challenging, to say the least. Attendant staff must respond to such behavior in a manner that is appropri*1365ate, reasonable and adequate. Given circumstances creating a substantial likelihood of physical harm to the patient or others, the Commonwealth, acting through hospital staff, may respond so as to ensure safety in the hospital community. The state’s parens patriae interest in protecting the safety of the people extends to the microcosm world of the hospital, as well as the community at large.

But, defendants’ proffered definition of an emergency justifying forced medication — the psychiatric emergency concept — is too broad, subjective and unwieldy. The fact that a set of circumstances may fall within the broad parameters of a psychiatric emergency does not necessarily justify any and all responsive steps taken thereafter by a doctor, even though therapeutic and well within the standards of reasonable medical practice.

This court holds, therefore, that a committed mental patient may be forcibly medicated in an emergency situation in which a failure to do so would result in a substantial likelihood of physical harm to that patient, other patients, or to staff members of the institution.

D. The Right to Refuse Treatment in a Non-Emergency

Given the uncontested right of the state to impose treatment without informed consent in an emergency, the court must now decide whether the state has a comparable right in a non-emergency. Because the state contends that the status of voluntary and involuntary patients is substantively different, their rights will be considered separately.

1. The Involuntary Patient’s Right to Refuse Treatment

As a matter of focus, it is important to bear in mind that in this case we are dealing with a hospital setting, not a jail. But, a mental hospital is unique in that its patient community is composed primarily of seriously disturbed persons, many of whom have the potential for dangerous behavior. And so whatever rights a patient may have in such a setting must be balanced with the needs and exigencies of the hospital community as a whole. Wolff v. McDonnell, 418 U.S. 539, 94 S.Ct. 2963, 41 L.Ed.2d 935 (1974).

The prime purpose of any hospital is to treat. Boston State is no exception. In the case of an involuntarily committed patient, Boston State has a duty to provide treatment. Stated another way, the involuntarily committed patient has a right to receive treatment. The question here is whether the hospital’s duty to provide necessary treatment carries with it an implicit right to impose such treatment contrary to a patient’s expressed wishes. In considering this question, it is important to have in mind that plaintiffs do not assert a right to refuse all treatment at all times. Their prime contention is that committed patients have a right not to be forcibly injected with psychotropic medication in a non-emergency situation, or where there are less drastic or less invasive alternatives available.

a. The Involuntary Patient’s Right to Privacy

This court has already found that psychotropic medications are powerful and potentially mind-altering drugs. See Sec. VII supra. Plaintiffs argue that the forcible injection, without informed consent, of such medication violates a patient’s constitutional right to privacy. They point out that the right of privacy has been broadly interpreted and applied in such diverse areas as marital relations,20 contraception,21 child rearing,22 possession of obscene material in the home,23 and bodily integrity.24

*1366Moreover, the Massachusetts Supreme Judicial Court has recognized that “in appropriate circumstances” a patient’s constitutional rights may include “the right of a patient to preserve his or her right to privacy against unwanted infringements of bodily integrity . . .” Superintendent of Belchertown v. Saikewicz, 373 Mass. 728, 370 N.E.2d 417, 424 (1977). Significant to the issue here is that court’s comment:

The constitutional right to privacy . . . is an expression of the sanctity of individual free choice and self-determination as fundamental constituents of life. The value of life as so perceived is lessened not by a decision to refuse treatment, but by the failure to allow a competent human being the right of choice.

370 N.E.2d at 426 (emphasis supplied).25

The defendants concede that a right to privacy may include “the interest in independence in making certain kinds of important decisions.” Whalen v. Roe, 429 U.S. 589, 599-600, 97 S.Ct. 869, 876, 51 L.Ed.2d 64 (1977). (Defendants’ Trial Brief p. 25). But, they argue that plaintiffs’ interest in refusing treatment in a mental institution setting is not a right fundamental to concepts of ordered liberty traditionally recognized and protected by the Supreme Court.26

The defendants’ position does not give due regard to the plight of a patient who has been committed to a state mental institution. We know that the committed mental patient has been quarantined from home, family and society, not for adjudged criminal activity, but because of sickness— mental illness.

The committed patient is in a foreign setting. He is in need of treatment, yet is presumed to be competent. Absent a successful petition by the hospital superintendent to establish a guardianship, we must assume that the hospital regards the patient as competent to manage his affairs.27

At final argument, the Commonwealth conceded that a committed patient would have the right to sell his home, but maintained that the patient has no rights with respect to what treatment to receive, if any, in a non-emergency situation. Common sense dictates a contrary conclusion, however. Certainly the right to dispose of one’s property,,and the corollary right to protect and hold such property, are fundamental to any concept of ordered liberty. See Lynch v. Household Finance Corp., 405 U.S. 538, 552, 92 S.Ct. 1113, 31 L.Ed.2d 424 (1972). But, such rights pale in comparison to the intimate decision as to whether to accept or refuse psychotropic medication — medication that may or may not make the patient better, and that may or may not cause unpleasant and unwanted side effects. The right to make such a decision is basic to any right of privacy.

b. The Involuntary Patient’s First Amendment Rights

The concept of a right of privacy also embodies First Amendment concerns.28 It is clear from the evidence in this case that psychotropic medication has the potential to affect and change a patient’s mood, attitude and capacity to think. Such effects may well be considered by the medical profession as positive steps on the road to *1367recovery and eventual release from the hospital. But, the validity of psychotropic drugs as a reasonable course of medical treatment is not the core issue here. At stake is the more fundamental question as to whether the state may impose once again on the privacy of a person, already deprived of freedom through commitment, by forcibly injecting mind-altering drugs into his system in a non-emergency situation.

The right to produce a thought — or refuse to do so — is as important as the right protected in Roe v. Wade to give birth or abort. Implicit in an individual’s right to choose either abortion or birth is an underlying right to think and decide. Without the capacity to think, we merely exist, not function. Realistically, the capacity to think and decide is a fundamental element of freedom.

The First Amendment protects the communication of ideas. That protected right of communication presupposes a capacity to produce ideas. As a practical matter, therefore, the power to produce ideas is fundamental to our cherished right to communicate and is entitled to comparable constitutional protection. Whatever powers the Constitution has granted our government, involuntary mind control is not one of them, absent extraordinary circumstances. The fact that mind control takes place in a mental institution in the form of medically sound treatment of mental disease is not, itself, an extraordinary circumstance warranting an unsanctioned intrusion on the integrity of a human being. The patient is in an institution only because he is unable to function safely in society, and so there is a public interest in civil commitment. The state may not involuntarily commit a person merely because of bizarre or unorthodox behavior.29

The makers of our Constitution undertook to secure conditions favorable to the pursuit of happiness. They recognized the significance
of man’s spiritual nature, of his feelings and of his intellect. They knew that only a part of the pain, pleasure and satisfactions of life are to be found in material things. They sought to protect Americans in their beliefs, their thoughts, their emotions and their sensations. They conferred, as against the Government, the right to be let alone — the most comprehensive of rights and the right most valued by civilized men.

The concept of a therapeutic alliance between doctor and patient presumes a communication of information as to the pros and cons of a particular treatment program. The committed patient has a right to be wrong in his analysis of that information — a right to be unwise — as long as the consequences of such error do not pose a danger of physical harm to himself, fellow patients or hospital staff. And so, while the state may have an obligation to make treatment available, and a legitimate interest in providing such treatment, a competent patient has a fundamental right to decide to be left alone, absent an emergency situation.30

2. The Voluntary Patient’s Right to Refuse Treatment

Defendants argue that voluntary patients may not refuse medication, even in non-emergencies, and still remain at the Hospital. Their position is that patients volunteering for commitment implicitly agree to accept the Hospital’s treatment program and may not second-guess the institution staff by picking and choosing the type of medication to be used. Basically, the defendants argue a contract theory that would supersede and amount to a waiver of any supposed right of refusal.

All voluntary patients sign an application that states:

“I understand that during my hospitalization and any after care, I will be given care and treatment which may include the injection of medicines.”

Four of the named plaintiffs (Bybel, Hunt, Rogers, and Warner) were, at times, volun*1368tary patients and so probably signed such a form. Presumably, they and other voluntary patients were competent to make the commitment decision — or someone was authorized to make that decision on their behalf. See Sec. VIII (A) supra.

Clearly, parties competent to contract may not accept provisions of the bargain they favor and then reject those they wish to avoid. But, such a truth serves only to raise the question presented here, not answer it. One remedy for the unhappy voluntary patient is clear, leaving the hospital. But procedures for doing so require a hiatus of three days’ notice from the patient to the hospital superintendent. See Sec. V supra. The issue, therefore, is really twofold: 1) what is the voluntary patient’s right to refuse treatment from the time of such notice to the time of departure from the hospital, and 2) even prior to notice, or absent notice, does the voluntary patient have a constitutional right to refuse treatment despite any contractual obligation that may have been established by the voluntary commitment?

This court holds that the voluntary patient has the same right to refuse treatment in a non-emergency as does the involuntary patient,31 and that on the facts of this case there has been no waiver of such right.

To support its waiver theory, the Commonwealth cites the Massachusetts case of Belger v. Arnot, 344 Mass. 679, 183 N.E.2d 866 (1962). In Belger, plaintiff claimed that certain involuntary electroshock treatments she received while hospitalized, pursuant to a temporary ten day admission, amounted to a battery. While there is language in the opinion which suggested that defendants were not liable because plaintiff had assented to all subsequent treatment, that case is inapposite to the one at bar.

First, the general issue before the Belger court was one of liability in tort, not the issuance of an injunction to protect constitutional rights. Second, the language here constituting application for voluntary commitment is neither a clear consent to be treated or a knowing voluntary waiver of a constitutional right to refuse treatment. At best, the language is ambiguous and could even be interpreted as a mere statement of expectation or entitlement by the patient.

In order for a court to find a waiver of a right to refuse, the evidence must be clear that the patient understood such a right existed and then elected knowingly and voluntarily to waive such a right.32 The language proffered by the defendants contains neither element.33 The Commonwealth drafted the language in the application and the defendants, as agents of the Commonwealth, must bear the burden of its inadequacy.34

E. The Commonwealth’s Interests

The Supreme Court has long recognized that fundamental rights are not absolute, but may be subordinated to compelling state interests. Roe v. Wade, supra, 410 U.S. at 155, 93 S.Ct. 705. The state has no such compelling interest here.

Each of us has a basic right to care for our bodies subject to “reasonable regulations, as the safety of the general public may demand.” Jacobson v. Massachusetts, 197 U.S. 11, 29, 25 S.Ct. 358, 862, 49 L.Ed. 643 (1905). Here, the state’s interest in protecting the safety of the general public is the justification for commitment of mental patients. After commitment, the balancing of state versus patients’ interests must be done in the context of the community setting.

*1369That the state may forcibly medicate a committed patient given an emergency that threatens the physical safety of patients and staff — the institutional community — is not at issue. Such action is necessary to protect the members of that community and is consistent with the basic rationale relied on in Jacobson. Given a non-emergency, however, it is an unreasonable invasion of privacy, and an affront to basic concepts of human dignity, to permit forced injection of a mind-altering drug into the buttocks of a competent patient unwilling to give informed consent. That type of treatment is not necessary to protect the general public, since the patient has already been quarantined by commitment. Of course, there being no emergency, the hospital community is in no danger.

The only purpose, therefore, of forced medication, in a non-emergency, is to help the patient. The desire to help the patient is a laudable if not noble goal. But, a basic premise of the right to privacy is the freedom to decide whether we want to be helped, or whether we want to be left alone. It takes a grave set of circumstances to abrogate that right. That a non-emergency injection in the buttocks may be therapeutic does not constitute such a circumstance.

Fourth Amendment cases cited by the defendants to support a state interest in forced medication are inapposite.35 The patients at Boston State are not subjects of criminal investigation, but are victims of fate who have been shortchanged by life. They have been isolated to protect the public, even though they are presumed to be competent to manage their affairs.

There are alternative methods of treating mental patients, though some may be slower and less effective than psychotropic medication. As has been noted, plaintiffs’ primary objection is to the forced injection of psychotropic medication. Given the alternatives available in non-emergencies, subjecting a patient to the humiliation of being disrobed and then injected with drugs powerful enough to immobilize both body and mind is totally unreasonable by any standard. Forced injections in non-emergencies are classic “intrusions which are not justified in the circumstances.” Schmerber v. California, 384 U.S. 757, 768, 86 S.Ct. 1826, 1834, 16 L.Ed.2d 908 (1966).

Aside from the proffered state interests in forced medication, the defendants also resist recognizing a right to refuse by challenging the efficacy of a regime of informed consent. The defendants’ crystal ball foretells a tale of gloom if the standards imposed by this court’s temporary restraining order (TRO) are made permanent. But, their prediction is not borne out by the experience of that order. Although Dr. Gill expressed concern as to the impact of the TRO on effective treatment, he could identify only 12 patients out of 1,000 who refused their medication for prolonged periods between May 1, 1975 and June 23, 1977 —and most of those changed their minds within a few days. None of these patients was transferred to a more secure institutional setting because of behavior problems.36

Should the TRO be made permanent, defendants foresee institutional settings becoming mere warehouses, characterized by increases in violence, patient apathy, length of stays and administrative problems. The evidence in this case, however, demonstrates that such a gloomy forecast is more dramatic than factual.

As has been pointed out, the target of plaintiffs’ suit is forced injection of psycho*1370tropic medication. Given a patient’s refusal, other traditional means of psychotherapy and treatment would continue to be available. The fact that some doctors may feel that the use of psychotropic drugs is quicker and more effective than other available treatment does not mean that patients should not be given the option to follow a more conservative course in a non-emergency setting. Also to be borne in mind is that the great majority of patients have not declined their psychotropic medication during the pendency of the TRO. Most of those who did changed their minds within a few days. This speaks well for the confidence in a doctor’s judgment that may be established given the effort to establish a strong therapeutic alliance.

The First Circuit recognizes an involuntary patient’s right to be secure from the assaults of fellow patients, Harper v. Cserr, 544 F.2d 1121 (1st Cir. 1976), and the state’s power “to establish reasonable involuntary psychiatric procedures to deal with highly disturbed persons having manifestly dangerous propensities.” Gomes v. Gaughan, 471 F.2d 794, 800 (1st Cir. 1973). Those propositions are not in dispute here, and this court’s TRO in no way impinged on the state’s power to act in such situations. In this regard, it is significant to recall the testimony of Dr. Gill to the effect that only 3% to 5% of the Austin Unit’s patients engaged in violent behavior, and only 20 to 25% of that population were even potentially violent.

In determining whether to order permanent injunctive relief, the issue before this court is not whether forced injection of psychotropic medication may be considered sound medical practice. Plaintiffs’ constitutional challenge raises the issues as to whether a patient may decline medication even though it may be beneficial, and whether that declination must be respected by hospital staff.

In an amicus brief, the Massachusetts Psychiatric Society argues:

If forbidden to use certain standard, effective modalities, they (hospital staffs) will be caught in the situation of having a legal obligation which they cannot carry out.37

(Amicus Brief pp. 4-5). That argument suffers from a faulty premise. The state has a duty to make treatment available. It has no duty to impose treatment on a competent involuntary patient who prefers to refuse medication, regardless of its potential benefit.

In analyzing defendants’ prediction of doom should the TRO be made permanent, it is helpful to recall the testimony of the defendants and most of their expert witnesses, to the effect that they would respect a patient’s preference to refuse treatment, absent an emergency situation. In other words, although not conceding a patient’s legal right to refuse treatment, the professionals’ practice is to honor the refusal, except in an emergency. That testimony is inconsistent with any prediction of a chaotic institutional atmosphere if patients’ wishes to refuse medication are honored. Certainly the expressed attitude of these interested professionals demonstrates that respecting and abiding by a competent mental patient’s wishes concerning medication would not undermine the ethical integrity of the medical profession.

The Commonwealth does have a legitimate interest in decreasing the number of patients hospitalized, as well as the length of their stays. One need only to be alive to be aware that the costs of illness, mental and physical, are soaring. There may well be additional administrative expense and burden attached to recognizing a competent inpatient’s right to refuse treatment. Stated another way, it might be less expensive for the state to deny, rather than recognize, such a right. But, factors of convenience and cost have long been regarded as inadequate justifications, standing alone, for a *1371state’s failure to recognize and respect constitutionally protected rights. Watson v. City of Memphis, 373 U.S. 526, 83 S.Ct. 1314, 10 L.Ed.2d 529 (1963); Rozecki v. Gaughan, 459 F.2d 6 (1st Cir. 1972).

One basic theme that seems to thread its way through most of defendants’ arguments is that a federal court has no business second-guessing a treatment decision of a hospital staff person. This contention is like saying that once there is confinement in prison there can be no judicial scrutiny as to the conditions of confinement. As Chief Judge Bazelon of the D.C. Circuit put it:

It makes little sense to guard zealously against the possibility of unwarranted deprivations prior to hospitalization, only to abandon the watch once the patient disappears behind hospital doors.

Covington v. Harris, 136 U.S.App.D.C. 35, 41-42, 419 F.2d 617, 623-24 (D.C.Cir.1969).

Professional judgments concerning confined persons, whether they be made by wardens in prisons or physicians in hospitals, should always be subject to judicial scrutiny, given an allegation of constitutional deprivation. Such an allegation has been made here, and it is well founded.38

The defendants are enjoined from forcibly medicating committed mental patients, voluntary or involuntary, except in emergency circumstances in which a failure to do so would bring about a substantial likelihood of physical harm to the patient or others. An order will issue.

IX. SECLUSION

Seclusion is the isolating of a mental patient in a small locked room. Plaintiffs contend that committed mental patients have a constitutionally protected right not to be secluded, except in emergency situations defined by M.G.L.A. ch. 123, § 21 and related DMH regulations.

Plaintiffs do not contend that seclusion is per se unconstitutional. They recognize that seclusion is a legitimate means for meeting emergency situations threatening the physical well-being of patients and staff. Their complaint is that seclusion was routinely used by defendants in non-emergencies for the purposes of treatment and punishment. Plaintiffs claim further that seclusion decisions, both to confine and release, were delegated by defendants to unauthorized staff personnel.

Defendants concede that M.G.L.A. ch. 123, § 21 is the controlling standard. They deny, however, that seclusion was used in other than emergency circumstances and, therefore, maintain there is no occasion for this court to grant the requested injunctive relief.

A. Seclusion Statutes and Regulations

Massachusetts permits the use of seclusion only in emergency situations “where there is the occurrence of [or] serious threat of extreme violence, personal injury, or attempted suicide.” DMH Reg. § 223.02; M.G.L.A. ch. 123, § 21. The statute requires that written authorization be obtained from the hospital superintendent or a designated physician prior to seclusion. If they should be unavailable, seclusion may be used, provided that such use is reported to the superintendent or physician within eight hours. The authorization form must also be signed by the person placing the patient in restraint. M.G.L.A. ch. 123, § 21; DMH Reg. § 223.05.

*1372A seclusion order is limited to eight hours and the secluded patient is to be visited at least once an hour by designated ward staff. DMH Reg. § 223.06. The superintendent or a designated physician is to review the use of seclusion at least once every eight hours and authorize in writing either its continuation or cessation. His determinations and reasons therefor are to be recorded on departmental form A32. DMH Reg. § 223.07. The case record of a secluded patient is to include the name of the person authorizing seclusion, the reason for seclusion, and the time that seclusion commenced and concluded. DMH Reg. § 223.-08. Violations of seclusion regulations are to be reported to the superintendent within 24 hours. DMH Reg. § 223.09.

On May 11, 1978, during the pendency of this suit, DMH issued a clarification of its seclusion policy. Under the clarification, emergencies are said to exist when there is substantial risk, or occurrence, of serious self-destructive behavior or physical assault. “Substantial risk” is defined as the serious, imminent threat of bodily harm combined with the present ability to inflict such harm. The term “self-destructive behavior” is not limited to suicide attempts.

Given an emergency situation, the least restrictive means of restraint is to be used. Behavioral symptoms clinically relied on as precursors of assaultive or self-destructive episodes are to be documented, as well as a description of any less restrictive alternatives that were utilized before seclusion.

Only the superintendent or a designated physician may order seclusion. Seclusion orders are valid for no longer than eight hours. Secluded persons are to be checked at least every 15 minutes. The staff person performing the check must indicate the time of his observation and also initial the appropriate box under the section marked “Safety Checks” on Form A-32-77.

The seclusion order form (DMH A-32-72) requires that the following information be provided when seclusion is used: a) the reason for seclusion, b) a description of the emergency situation, c) the name and signature of the ordering physician, d) the name and signature of the person applying the seclusion, e) the commencement and conclusion time, f) the name of the person removing the seclusion.

The form also states that anyone ordering or applying restraints or seclusion must be familiar with the applicable laws and regulations set forth on the reverse side of the form. On the other side of the order form is a verbatim statement of M.G.L.A. ch. 123, § 21, a description of seclusion record-keeping requirements, and a summary of procedures to be used in handling violations of seclusion regulations. Also on the back of the order form is a DMH policy statement that seclusion is to be utilized as a last resort.

B. Seclusion Facilities at the Austin and May Units

Although they varied somewhat, the seclusion facilities at the Austin and May Units had comparable characteristics. They were small rooms, approximately six by twelve feet, unfurnished except for a mattress and cover. The floors were bare. There was an overhead light. Ventilation and heating were erratic, though not dissimilar from that in the rest of the unit. Temperature control was a chronic problem. Secluded patients were disrobed, except for underwear and a hospital gown. Often patients would remove even these garments.

Each room contained a single window that was locked and screened. The door contained a small screened window that permitted a limited view of the interior. Patients would often relieve themselves in the seclusion room and so, despite sanitation efforts, strong odors of urine and feces persisted. The rooms were chronically in need of repair and cleaning.

While secluded, patients were isolated from outside contact. They were not permitted to read, write, have visitors, or participate in any recreational, therapeutic or educational activity on the ward.

*1373On the basis of the evidence presented, and a view of the Austin and May Units, this court finds that the seclusion rooms during most of the relevant period were dirty, stark, austere and smelly. They were unpleasant places to be confined, to say the least. But, even with their deficiencies, they did not have a dungeon-like atmosphere that would shock the conscience, thereby triggering Eighth Amendment considerations of cruel and unusual punishment. They, at least minimally, met the requirements of DMH Reg. § 223.-04(b),39 and had the capacity to effectively isolate an out-of-control patient from other patients and staff, for the protection of all concerned.

C. Seclusion Practices at the Austin and May Units

Between January 1, 1973 and April 25, 1975, seclusion was routinely used at both the Austin and May Units in a variety of circumstances that did not constitute an emergency as defined by M.G.L.A. ch. 123, § 21. Moreover, seclusion was used, during that period, in circumstances where a less restrictive alternative would have been adequate to terminate any existing emergency had additional staff and resources been available at the hospital.

Staff doctors commonly authorized the seclusion of patients “PRN.” A doctor’s PRN order has the effect of authorizing seclusion in the discretion of non-physician staff. Basically, a PRN order permitted staff to seclude “as necessary,” any time of day or night, without having to consult a physician first. At times the PRN seclusion orders preceded by as many as 24 hours the incident that resulted in seclusion. When seclusion was ordered PRN, patients were not required to be examined psychologically or physically, either prior or subsequent to the seclusion.

Although the seclusion sheet required that observations be detailed, usually only a general description of the emergency requiring seclusion was given. The form often remained unsigned by a physician for several weeks after the patient had been secluded.

1. The Austin Unit

Prior to this court’s temporary restraining order, the doctors and staff at the Austin Unit used seclusion to extinguish bizarre but non-dangerous patient behavior. Dr. Gill’s policy at Austin was to permit seclusion when he felt it would be useful in the treatment of a patient. Routinely, he and the Austin staff used seclusion as negative reinforcement to modify patient behavior they deemed to be undesirable. An example is the treatment plan of “B.M.” which included “a behavior modification program on ward with use of seclusion privilege for reward/punishment.”

After this court’s temporary restraining order, Dr. Gill directed that patients no longer be secluded as part of treatment plans. This was a change in policy and practice at the Austin Unit.

2. The May Unit

The May Unit also followed a policy of secluding for treatment purposes in non-emergency situations. For example, patient Sam D. was secluded each evening to prevent him from stealing from other patients. His evening seclusion was a matter of standing order. Patient Bunny R. had a treatment plan that required her to be placed in seclusion for 15 minutes when testy, or if she poked or pushed someone, although she never hit anyone. Patient Richard W. was automatically secluded for escaping. Two patients, Gail J. and Al U., were secluded for engaging in sexual relations. Patient Everett A. had a treatment plan calling for seclusion if he escaped, or would not take medication, or made verbal threats to staff. Patient Rita F. was secluded for walking nude in the day hall. Patient Barbara R. was secluded for refusing to stop talking loudly and saying *1374negative things about herself. Patient David S. was secluded for masturbating in the day hall.

Hospital records relating to seclusion of patients at the Austin and May Units are voluminous, and it would be impractical to outline them here. It is sufficient to note this court’s finding that the records of Austin and May demonstrate unequivocally that seclusion was routinely used in connection with treatment plans to modify behavior that could not reasonably be considered “a serious threat of extreme violence, personal injury, or attempted suicide.” M.G. L.A. ch. 123, § 21. This court finds, however, that in those instances of misuse, seclusion was employed at Austin and May as a treatment modality and not as punishment.

D. Discussion

Applying the standards of Mathews v. Eldridge, 424 U.S. 319, 96 S.Ct. 893, 47 L.Ed.2d 18 (1976), defendants claim that their seclusion practices served to protect the “private interest,” 424 U.S. at 335, 96 S.Ct. 893, of the patients of the Austin and May Units. They take the position that, “[i]t is actually in the self-interest of the patient to allow staff to protect him during those times the patient is out of control . . [T]he state has a heavy interest in swiftly controlling the violent or potentially violent patient for his or her protection and for the protection of the other patients.” (Defendants’ Trial Brief p. 62).

But, whatever merit there might otherwise be in defendants’ position, their faulty factual premise makes it inapposite to the Boston State situation. There were certainly many instances presented at trial of seclusion decisions having been made in response to emergency situations threatening the safety of patients and staff. As has been pointed out above, however, the great weight of evidence demonstrates that, prior to this court’s restraining order, seclusion was used routinely as a treatment modality, and not merely as a procedure for restraining patients who were out of control. Such a finding is supported not only by the weight of the testimony presented, but by the Hospital’s records that were received in evidence.

Defendants ask the court to ignore the clear message of those records on the grounds that the references to seclusion as treatment were either mistakes or misnomers. But, for this court to do so would be to deny the presumed reliability of “routine, standard, and unbiased medical reports by physician specialists.” Mathews v. Eldridge, supra, 424 U.S. at 344, 96 S.Ct. at 907. Richardson v. Perales, 402 U.S. 389, 404, 91 S.Ct. 1420, 28 L.Ed.2d 842 (1971). The bulk of these records were prepared as a matter of hospital routine long before this litigation commenced. Whatever deficiencies they may have in terms of clarity and completeness, these records do serve the purpose of clearly corroborating the testimony of those witnesses who testified that seclusion was routinely used at Boston State for treatment and not merely as an emergency restraint.

Plaintiffs have a right to the protections afforded by the laws and regulations of the Commonwealth, the constitutionality of which is not in issue. Defendants have an obligation to provide that protection and, therefore, are enjoined from applying seclusion to committed mental patients except in emergency situations where there is the occurrence or serious threat of extreme violence, personal injury, or attempted suicide. The defendants are also enjoined from violating related Massachusetts law concerning seclusion, including state statutes and D.M.H. regulations.

In view of the court’s holding, it is unnecessary to reach plaintiffs’ constitutional claims in considering there request for injunctive relief against defendants’ seclusion practices. But, it is necessary to decide these constitutional claims as a prelude to considering plaintiffs’ federal damages claims under 42 U.S.C. § 1983. (Sec. X(B)(1) infra.)

Defendants claim that there was no due process violation here “since procedures had been established in the Austin and May *1375units to ensure that the seclusion decision is made only in the patient’s best interest.” (Defendants’ Trial Brief p. 64). But, plaintiffs are entitled to greater protection than defendants’ proffered “best interest” test. Plaintiffs had a right under Massachusetts statutory law, M.G.L.A. ch. 123, § 21, not to be secluded except in a clearly defined emergency. Defendants had a corollary responsibility to protect their patients from a misuse of seclusion.

Defendants’ reliance on Meachum v. Fano, 427 U.S. 215, 96 S.Ct. 2532, 49 L.Ed.2d 451 (1976), overlooks the fundamental factual distinction between that case and this. There, an inmate opposing an interprison transfer was unsuccessful because “Massachusetts law conferred no right of the prisoner to remain in the prison to which he was initially assigned.” 427 U.S. at 226, 96 S.Ct. at 2539. Here, however, there is a statutory right not to be secluded except in response to an emergency. Moreover, the Commonwealth’s regulations established procedures designed to ensure that plaintiffs would receive the protection afforded them under M.G.L.A. ch. 123, § 21. The indiscriminate use of seclusion PRNs, the failure to properly fill out seclusion order and observation forms, and the failure to review incidents of seclusion within eight hours, were all violations of Massachusetts law. The court, therefore, holds that the defendants' actions with respect to seclusion violated plaintiffs’ due process “liberty interest” under the Fourteenth Amendment.

X. NAMED PLAINTIFFS’ CLAIMS FOR DAMAGES

In addition to seeking injunctive relief, the named plaintiffs claim they are each entitled to an award of damages because of defendants’ medication and seclusion practices. They seek such recovery under a variety of theories embodying both federal and state causes of action.

Defendants oppose any award of damages. First, they deny any lack of due care in their treatment of plaintiffs. Second, they assert that they acted in good faith as state employees and, therefore, are immune from liability.

This court has already determined that, at one time or another, the defendants countenanced and routinely implemented policies of forced medication and involuntary seclusion of patients at Boston State in non-emergency circumstances when there was no substantial likelihood of physical harm to patients or others. Such general findings, though adequate for injunctive relief, are not sufficient to support individual damage claims by the named plaintiffs. Rather, it is necessary that each named plaintiff establish, by a preponderance of the evidence, that he or she was damaged because of defendants’ impermissible seclusion or medication practices. And so, prior to analyzing the merits of plaintiffs’ asserted theories of recovery, it is necessary for this court to make findings as to the factual bases of their claims.40

A. Findings of Fact Related to Damages Claims

1. May Unit Plaintiffs

The May Unit plaintiffs are Harold Warner, Able Bolden, Willie Wadsworth and Rubie Rogers. They claim that they were involuntarily medicated and secluded in non-emergency circumstances. First, the court examines their medication claims.

a. Medication

This court finds that only two May Unit plaintiffs — Rubie Rogers and Willie Wads-worth — were forcibly medicated in non-emergency circumstances.

Rogers refused medication throughout her hospitalization. Her refusals were respected only when she was in “reasonable control.” But, the only occasions in which she was sufficiently out of control to warrant seclusion occurred in *1376September, 1973 and June, 1974. Given Rogers’ innumerable refusals, the May Unit’s policy to forcibly medicate in non-emergencies, and the fact that she lived on the ward for several months without requiring seclusion, this court draws the inference, and so finds, that Rogers was involuntarily medicated in circumstances that would not constitute an emergency under M.G.L.A. ch. 123, § 21.

Wadsworth was secluded for 30 days, commencing in December of 1974. While secluded, he was forcibly medicated. Since Wadsworth was already restrained by seclusion, his refusal to take medication would not have precipitated an emergency.

The court finds that Bolden and Warner were not forcibly medicated in non-emergencies. While I.M. medication was prescribed for Bolden, the court is not persuaded that these orders were carried out in non-emergencies. Rather, the evidence supports a finding that medication was forced on Bolden, if ever, only when he was acutely psychotic and out of control.

The only evidence that Warner was involuntarily medicated was through his testimony. The court does not find that testimony to be credible.

The court is also persuaded that Rogers suffered some side effects from her medication, including akathisia and akinesia, terms that have already been defined. See Sec. VII supra. Her anti-psychotic medication was terminated when these side effects were recognized by staff.41

b. Seclusion

The evidence demonstrates that three of the named plaintiffs — Wadsworth, Warner and Bolden — were impermissibly secluded in non-emergency situations.

Wadsworth, a prior patient, was readmitted to the May Unit on Christmas Day, 1974, in an acutely psychotic condition. In addition to punching an attendant in the chest, Wadsworth rolled a coat hanger around his fist and threatened to use it if an attempt were made to seclude him. After being confronted by a number of attendants, Wadsworth went into seclusion on his own, where he spent the next 30 days.

During that time, the staff used lithium on him in an effort to control what they perceived to be his manic condition, and a number of seclusion programs were fashioned for him. After the fifth day of seclusion, his seclusion program permitted four thirty-minute time out periods per day. On the tenth day of seclusion, that time out period was increased to one hour. On the sixteenth day, he was released for one and one-half hours, six times a day, and was permitted to eat in the day hall. By the twenty-second day, he was permitted to be out of seclusion for two hours at a time.

There is little question that the first four or five days of Wadsworth’s seclusion were justified. He was a relatively big and powerful young man, exhibiting hostile and assaultive behavior at the time of his admission to the May Unit.42

While the initial seclusion was justified, the twenty-five days of seclusion that followed were not. Progress notes indicate that, for most of that time, Wads-worth was in control.43 By the eighth day, the night staff felt that he no longer needed to be secluded. Of the approximately two thousand entries on seclusion room observation sheets, from December 28,1974 to January 23, 1975, only six indicated any *1377agitated behavior. Three of these were on December 30, 1974.44

The fact that Wadsworth’s treatment program permitted increasing hours out of seclusion was not a per se violation of M.G. L.A. ch. 123, § 21. Observations made during such “time out” periods may well serve as a barometer of a patient’s ability to handle the stimulus of the ward. But, this court finds that Wadsworth’s consistent non-violent behavior during time out periods demonstrated that he did not need to be secluded.45

Only five seclusion orders were filled out for Warner. This court finds that the seclusion order of March 22, 1975, was not prompted by an emergency. Hospital records for that day indicate that he was “creating a disturbance on other wards” and that he was “obnoxious, instigating trouble with patients and staff.” Of significance is that a February 24 entry in the seclusion order book instructed that Warner be secluded for twelve hours for being a “loudmouth after warnings.” The inference is warranted that the March 22nd seclusion was for disruptive behavior short of an emergency46 and this court so finds.

Able Bolden was secluded approximately sixty times between December 1974 and April 1975. The bulk of these seclusions occurred in February and March of 1975, while he was acutely psychotic and out of control.47 Nonetheless, the court is persuaded that one of Bolden’s seclusions was in violation of M.G.L.A. ch. 123, § 21. On March 4, 1975, Bolden was to be secluded for twenty-four hours as a result of having sex with another patient. The Hospital’s records indicate that he was taken out of seclusion for a neurological appointment at Shattuck Hospital, but was to be secluded again for the balance of the twenty-four hours after his return. The inference is warranted that Bolden would not have been taken to a neurological examination if he were an emergency threat, and that his seclusion upon return from Shat-tuck was unrelated to any emergency.48 The court therefore finds that the seclusion was in violation of M.G.L.A. ch. 123, § 21.

The court finds that the plaintiff Rogers has not sustained her burden of proving that any of her seclusions was in violation of the law.

2. Austin Unit Plaintiffs

Three Austin Unit patients are named plaintiffs in this suit — Donna Hunt, Elizabeth Bybel and James Colleran. All have medication and seclusion damages claims.

*1378a. Medication

This court finds that Hunt, Bybel and Colleran were forcibly medicated in non-emergency situations as part of a psychotherapeutic program.49

It is also apparent that Bybel and Hunt suffered side effects from the medication they received. Bybel received large dosages of phenothiazines during certain periods of her hospitalization, particularly in late January and February of 1974. There was evidence that she suffered from slow thinking and slurred speech following that medication. Hunt also experienced certain ex-trapyramidal effects from her heavy doses of medication, including akinesia. While one Austin Unit doctor felt that Hunt exhibited early signs of tardive dyskinesia, this court is not persuaded that she ever contracted the disease.

b. Seclusion

Perhaps the most contested issue in this case concerns the seclusion programs that were instituted for Donna Hunt. At the time of her admission to the Austin Unit on January 15, 1975, Hunt was a fifteen year old moderately retarded adolescent. Like her two Austin Unit co-plaintiffs, she had a capacity for violence and out-of-control behavior.50 In an effort to control that behavior, the staff decided to seclude her. During the next fifteen months, she was secluded for more than 1800 hours.

During Hunt’s first six weeks of hospitalization, she was slowed down by her body cast and physical illness. When the cast was removed, her behavior became assaultive. In early March, a program was implemented whereby Hunt was to be secluded for a minimum of four hours after each assaultive outburst. A staff note, dated April 9, 1974, summarized the treatment program as a “mild behavior modification plan to reduce incidents of striking people.” That program was unsuccessful, because Hunt seemed to enjoy the attention the seclusion process focused on her. Consequently, in an attempt to make the seclusion room a less desirable place, a plan was implemented calling for twenty-four hour seclusion following each assaultive outburst. Under this program Hunt could not be released until the twenty-four hour period expired, regardless of her apparent condition.

Shortly after its inception, this program was felt to be ineffective because Hunt’s agitation and self-destructive behavior increased even while in seclusion. That program was replaced, therefore, by one requiring an eight hour seclusion. Under this program, Hunt could reduce her required seclusion time by one hour for each day she remained out of seclusion. But, her self-destructive behavior increased, and so this program was replaced by one requiring constant seclusion with one hour of time out during each of the three staff shifts. Her *1379time out could increase by one hour per shift if she handled it well.

As of July 1974, a new program was instituted whereby Hunt was required to spend one hour of the day and evening shifts in her room to reduce stimulation and prevent her from engaging in assaultive and self-destructive acts. If she would not spend the time in her room, the program required that she spend it in seclusion. She was secluded less during July and August than during the two prior months.

Hunt’s mother came to visit her at the end of August of 1974. That visit had an adverse effect. According to hospital records, Hunt was out of control 98% of the time thereafter. In response, the Austin Unit staff reinstituted the twenty-four hour seclusion program that allowed increased time out for good behavior.

An October 1974 progress note indicated that it took Hunt the better part of the month to work her way out of seclusion. From that point on, Hunt was no longer secluded for fixed periods, but was required to spend one hour a shift in her room, or face seclusion. By November of 1974, she was able to remain out of seclusion most of the time. From then through February, 1975, her seclusion time decreased steadily to the point where she was secluded only forty hours during February.

While Hunt was chronically out of control during much of 1974,51 this court is persuaded that she was secluded in situations that did not pose a serious threat of extreme violence, personal injury, or attempted suicide. In those situations, seclusion was used as a treatment modality and not a restraint.52

Like Donna Hunt, James Colleran was admitted to the Austin Unit at the age of fifteen. From October of 1973 through April of 1975, he was secluded a total of forty-five times. A substantial number of those seclusions revolved around his sexual behavior. Colleran could be extremely assaultive when he was sexually agitated— engaging in persistent touching, pinching and fondling of both male and female patients. On other occasions, he would expose his genitals on the ward. In the spring of 1975, this behavior escalated to the point where Colleran was consistently subject to assault by patients who were provoked by his unsolicited sexual advances.

*1380Colleran maintains that the Austin Unit staff utilized seclusion as a behavior modification to extinguish his sexual behavior. One of Colleran’s treatment plans stated that he was to be secluded for “a period not to exceed three hours for exposure of sexual organs and/or overt contact with female staff and patients — if verbal request to discontinue such acting out is not responded to.” This court is persuaded that Colleran’s sexual aggressiveness constituted an emergency situation at the Austin Unit.53

The plaintiff Bybel was secluded for more than one thousand hours from January 1,1973 to April 30,1975. The bulk of these seclusions, and the ones to which most of the testimony was directed, occurred in two separate periods: April of 1974 (138 hours) and March and April of 1975 (575 hours). This court finds that Bybel was out of control during much of her stay at the Austin Unit,54 but that she was occasionally secluded in non-emergencies.55

B. Legal Conclusions Related to Damages Claims

1. Plaintiffs’ Federal Claims

Before analyzing the specifics of plaintiffs’ federal claims, it is useful to consider the essential elements they must prove, as well as defenses that may be available to meet such claims.

Any person who acts under col- or of state law to deprive another of a constitutional right may be required to compensate that person by an award of money damages. 42 U.S.C. § 1983.56 This court has already determined that all committed mental patients have a constitutional right not to be forcibly medicated or secluded except in a clearly defined emer*1381gency situation. In order to recover under § 1983, each of the named plaintiffs must demonstrate, by a preponderance of the evidence, a deprivation of at least one of these constitutionally protected rights, and that such deprivation was the proximate cause of injury to that plaintiff.

Given a demonstration of personal constitutional deprivation, a plaintiff may be entitled to an award of compensatory damages for any physical or emotional harm caused thereby, as well as an award for all pecuniary loss. Magnett v. Pelletier, 488 F.2d 33 (1st Cir. 1973). In addition, exemplary or punitive damages may be awarded if the trier of fact is persuaded, by a preponderance of the evidence, that a defendant’s actions amounted to a demonstration of improper motive, outrageous conduct and overall bad faith. Caperci v. Huntoon, 397 F.2d 799 (1st Cir.), cert. denied, 393 U.S. 940, 89 S.Ct. 299, 21 L.Ed.2d 276 (1968); Mansell v. Saunders, 372 F.2d 573 (5th Cir. 1967).

The character of defendants’ conduct has relevance beyond the issue of a possible award of exemplary damages, however. Indeed, a persuasive demonstration of good faith by a defendant may entitle him to qualified immunity from any award of money damages, despite the fact that his actions caused a constitutional deprivation.

These considerations suggest that, in varying scope, a qualified immunity is available to officers of the executive branch of government, the variation being dependent upon the scope of discretion and responsibilities of the office and all the circumstances as they reasonably appeared at the time of the action on which liability is sought to be based. It is the existence of reasonable grounds for the belief formed at the time and in light of all the circumstances, coupled with good-faith belief, that affords a basis for qualified immunity of executive officers for acts performed in the course of official conduct.

Scheuer v. Rhodes, 416 U.S. 232, 247-48, 94 S.Ct. 1683, 1692, 40 L.Ed.2d 90 (1974).

In Wood v. Strickland, 420 U.S. 308, 95 S.Ct. 992, 43 L.Ed.2d 214 (1975), a suit brought by students against high school officials, the Supreme Court defined the phrase “good faith” in terms stressing both objective and subjective components:

The official himself must be acting sincerely and with a belief that he is doing right, but an act violating . . . constitutional rights can be no more justified by ignorance or disregard of settled, indisputable law . . than by the presence of actual malice.
. A compensatory award will be appropriate only if the school board member has acted with such an impermissible motivation or with such disregard of the student’s clearly established constitutional rights that his action cannot reasonably be characterized as being in good faith.

420 U.S. at 321, 322, 95 S.Ct. at 1000-01.

Although the “good faith” definition enunciated in Wood embodies both objective and subjective standards, mere good intentions or “[sjubjective good faith in one’s conduct is not sufficient to protect an official from civil rights liability.” Perez v. Rodrigues Bou, 575 F.2d 21, 23 (1st Cir. 1978). The test is whether defendants “knew or should have known” that their actions would violate plaintiffs’ constitutional rights. Wood v. Strickland, supra, 420 U.S. at 322, 95 S.Ct. 992; O’Connor v. Donaldson, 422 U.S. 563, 577, 95 S.Ct. 2486, 45 L.Ed.2d 396 (1975).57

Further guidance on the availability of a good faith defense for actions taken in a mental hospital setting is provided by the recent First Circuit case of Downs v. Sawtelle, 574 F.2d 1 (1st Cir.), cert. denied, 439 U.S. 910,99 S.Ct. 278, 58 L.Ed.2d 255 (1978). There, defendant doctors were charged with conspiring to sterilize a deaf mute without her consent.

*1382[I]f a jury could reasonably conclude that Dr. Curtis determined that sterilizing the plaintiff was for her own good or the good of society and as a consequence of that belief ignored indications from the plaintiff that she did not consent to the operation, or if it could conclude that he attempted to take advantage of her mental and communication limitations to unduly influence her decision, he would be liable under both components of the Wood test. He should reasonably have known that such conduct amounted to an unconstitutional deprivation and he would be acting with a malicious motive. The fact that the doctor thought he had plaintiff’s best interests at heart would not justify a qualified immunity for constitutional purposes any more than would the belief, if asserted by a discriminatory employer or educator, that minority group members are happier and more productive in a segregated environment.

574 F.2d at 12 (footnote omitted).

In determining the validity of defendants’ proffered good faith defense, this court must consider what they “knew or should have known” with respect to plaintiffs’ constitutional rights in relation to the state of the law during the relevant period of 1973 to 1975. Haines v. Kerner, 492 F.2d 937, 941 (7th Cir. 1974); Hostrop v. Board of Jr. College, Dist. No. 515, 523 F.2d 569, 577 (7th Cir. 1975), cert. denied, 425 U.S. 963, 96 S.Ct. 1748, 48 L.Ed.2d 208 (1976). The defendants are not charged with a duty to anticipate then uncharted constitutional developments. Wood v. Strickland, supra, 420 U.S. at 322, 95 S.Ct. 992.

In addition, the court must evaluate the actions of the defendants in the context of any exigencies that existed on the wards of the Boston State Hospital. These include the adequacy of facilities and staff, as well as the potential for disruptive, if not violent, behavior at that institution. See Glasson v. City of Louisville, 518 F.2d 899, 908 (6th Cir.), cert. denied, 423 U.S. 930, 96 S.Ct. 280, 46 L.Ed.2d 258 (1975).

Any evidence as to defendants’ motive in forcibly medicating or secluding plaintiffs must be weighed. For example, it would be relevant to consider whether the evidence demonstrated that a plaintiff was either forcibly medicated or secluded as an act of punishment, vengeance, or even sadism. On the other side of the coin, it would be relevant, although not controlling, see Downs v. Sawtelle, supra, that the defendants’ sole motive was to treat and help the affected plaintiff.

Consideration should be given as well to the defendants’ conduct in relation to the plaintiffs’ helplessness and dependency. Callous or wanton neglect may warrant a finding of malice, as may reckless indifference, Harper v. Cserr, supra, 544 F.2d at 1125; Kelley v. Dunne, 344 F.2d 129, 133 (1st. Cir. 1965) (quoting Vigoda v. Barton, 348 Mass. 478, 204 N.E.2d 441 (1965)). A finding of wanton or reckless behavior may be based in part on a determination that the victim was helpless. Duchesne v. Sugarman, 566 F.2d 817, 828 (2d Cir. 1977).

Application of a good faith defense implies that, at the least, a mistake was made — something went wrong. The evolution of the defense is in part due to a recognition that the public interest is best served by enabling officials in challenging positions, such as the defendants here, to perform their duties with discretion, but without intimidation. This means that a monetary award of damages should not be imposed “for mistakes which were not unreasonable in the light of all the circumstances . . . .” Wood v. Strickland, supra, 420 U.S. at 319, 95 S.Ct. at 999.

Having these standards in mind, this court determines that defendants’ good faith immunity defense to plaintiffs’ § 1983 claims is valid. The defendants desired only to help plaintiffs. Their seclusion and medication practices were intended as treatment, not punishment. The prime constitutional issue here — the right to refuse treatment — was not one that a doctor was bound to anticipate in 1973. Moreover, that practice received tacit support from the Su*1383preme Judicial Court in 1968 and had never been ruled unconstitutional.58

Of significance to this court, as well, are the less than desirable conditions in which defendants were required to deal with plaintiffs. The facilities and support staff at Boston State were marginal, at best. In contrast, the patient population was extremely demanding, both in terms of numbers and their potential for disruptive behavior. Defendants did not have the luxury of detached, leisurely reflection as they faced the innumerable crises that characterized daily living on the Austin and May wards. They met those crises decisively, with the purpose of restoring plaintiffs to self control. To hold them liable in damages, given the totality of the circumstances presented by the evidence in this case, would be unfair to them and contrary to the public interest.

2. Plaintiffs’ State Claims

a. Assault and Battery, False Imprisonment

Plaintiffs maintain a right to recover damages under the intentional tort theories of false imprisonment and assault and battery. The latter requires proof of an unconsented touching, however slight, that is either harmful or offensive. The former consists of the unlawful restraint of a person’s freedom of movement.

This court assumes as inevitable the fact that the named plaintiffs, while institutionalized, were either touched or restrained against their wills. Plaintiffs argue that the traditional common law elements of these intentional torts are applicable to such occasions. This court disagrees.

Common sense, if nothing else, argues against the literal application of these torts to the activities and environs of a state mental hospital. To impose liability on physicians for good faith, non-negligent touchings and restraints would impede if not immobilize the administration of such an institution.

While there is no Massachusetts ease directly on point,59 a number of other jurisdictions have recognized the reality of the situation and have rejected the notion that traditional intentional tort concepts apply to the care of the institutionalized mentally ill. In the case of O’Donoghue v. Riggs, 73 Wash.2d 814, 440 P.2d 823, 828 n. 2 (1968), the Supreme Court of Washington held:

One who enters a hospital as a mentally ill person either as a voluntary or involuntary patient, impliedly consents to the use of such force as may be reasonably necessary to the proper care of the patient or for the necessary enforcement of reasonable rules governing the patient’s safety and health. ... So, also, the reasonable use of force may be necessary within a mental hospital in the proper care and treatment of a patient and in order to protect him or others from harm.

440 P.2d at 828 n. 2.

Other jurisdictions have also recognized the law of malpractice to be more appropriate than intentional tort theories in deciding claims arising at mental hospitals. In Hammer v. Rosen, 7 N.Y.2d 376, 198 N.Y. S.2d 65, 165 N.E.2d 756 (1960), the court held that treatment challenged there, *1384though clearly a battery, constituted a prima facie case of malpractice that could be rebutted by evidence that such treatment was proper. 198 N.Y.S.2d at 67, 165 N.E.2d at 757. In Morgan v. State, 40 A.D.2d 891, 337 N.Y.S.2d 536 (1972), a damage award to an involuntarily committed patient who had been tied to her cot for several weeks was reversed because there was “no medical testimony that the actions involved were improper.” 337 N.Y.S.2d at 537.

This court concludes, therefore, that the traditional elements of these intentional torts should not be the standard by which the defendants’ conduct is judged. Rather, determination of defendants’ liability should be based on a comparison of their conduct with standards of reasonable medical practice. The Belger case, supra, indicates that the Supreme Judicial Court would take such an approach if this issue were presented to it.

b. Malpractice

In order to sustain their medical malpractice claim, plaintiffs must prove the defendants’ negligence by establishing: 1) a duty of due care owed them by defendants, 2) the breach of that duty, and 3) they were damaged as a proximate result of that breach.

The standard of care owed by a physician to a patient in Massachusetts was defined by the Supreme Judicial Court in the case of Brune v. Belinkoff, 354 Mass. 102, 235 N.E.2d 793 (1968):

The proper standard is whether the physician, if a general practitioner, has exercised the degree of care and skill of the average qualified practitioner, taking into account the advances in the profession. In applying this standard it is permissible to consider the medical resources available to the physician as one circumstance in determining the skill and care required. Under this standard some allowance is thus made for the type of community in which the physician carries on his practice. See Prosser, Torts (3d ed.) sec. 32 (pp. 166-167); compare Restatement 2d: Torts, Sec. 299A, comment g-
One holding himself out as a specialist should be held to the standard of care and skill of the average member of the profession practicing the specialty, taking into account the advances in the profession. And, as in the case of the general practitioner, it is permissible to consider the medical resources available to him.

354 Mass, at 109, 235 N.E.2d at 798. As with other negligence actions, the damages that are compensable in a malpractice case include past and future pain and suffering, medical expenses, diminution in earning capacity, and loss of consortium. See Cuddy v. L & M Equipment Co., 352 Mass. 458, 225 N.E.2d 904 (1967); Rodgers v. Boynton, 315 Mass. 279, 280, 52 N.E.2d 576, 577 (1943); Donoghue v. Holyoke St. Ry. Co., 246 Mass. 485, 141 N.E. 278 (1923); Cochran v. City of Boston, 211 Mass. 171, 97 N.E. 1100 (1912).

In determining whether any of the defendants was negligent, the reasonableness of their conduct must be weighed in the context of the circumstances and surroundings in which they practiced. It is relevant, therefore, to consider the medical resources and support facilities available to the defendants at the Boston State Hospital. Brune v. Belinkoff, supra, 354 Mass, at 109, 235 N.E.2d 793.

These resources were barely adequate. For most patients, Boston State was the end of the treatment line. Its prime function was to deal with the most disturbed and potentially violent patients, those for whom local mental health clinics could not care. Despite this most challenging assignment, salaries at Boston State, and other state institutions, were not competitive with private institutions such as McLean’s Hospital. It is not surprising, therefore, that the Commonwealth found it difficult to attract and retain competent and experienced staff.60

*1385In addition to staffing deficiencies, the May and Austin physical plants were anything but models. Their heating systems were archaic and unreliable. Seeping steam caused cracked and peeling paint. Inadequate lighting cast a gloomy pall. Their horse trough type bathing facilities were termed “Dickensian” by Dr. Gill.

Naturally, the quality of care that May and Austin doctors were able to provide was affected by such factors. But, defendants had little control over the quality or quantity of staff or physical resources. Like front line surgeons, they were required to work with what they had. The court concurs with the position of the American Psychiatric Association that it would be unjust and unreasonable for courts to hold psychiatrists personally and individually responsible for resource deficiencies that are actually the responsibility of society.61 Such a decision would only deter qualified psychiatrists from working in the very setting where they are most needed.

Except in the unusual case in which “ ‘the negligence and harmful results are sufficiently obvious to lie within common knowledge,’ ” the elements of a medical malpractice case must be proved by expert testimony. Haggerty v. McCarthy, 344 Mass. 136, 139, 181 N.E.2d 562, 565 (1962) (quoting Cyr v. Giesen, 150 Me. 248, 252, 108 A.2d 316, 318 (1954)); McCarthy v. Boston City Hospital, 358 Mass. 639, 643, 266 N.E.2d 292, 295 (1971). This case is no exception. The complex psychiatric and medical issues involved are beyond the “common knowledge” of this court. The plaintiffs, therefore, must establish by expert testimony the average degree of care and skill exercised by qualified practitioners in comparable circumstances, and the fact that defendants deviated from such standards.

There was conflicting expert testimony as to the reasonableness of the medical treatment given the named plaintiffs by defendants. This court permitted plaintiffs to introduce testimony of non-psychiatrists on the issue of defendants’ lack of due care. The court deems such evidence to have been competent, particularly with respect to medication issues.

The proper use of medication and the predictable consequences of medication are matters not peculiar to the specialty of psychiatry. The fact that medication is used at a mental institution does not make the subject any more parochial. The same is true with seclusion practices and their predictable consequences. Seclusion is not an exclusive preserve of the psychiatrist, any more than is medication. Indeed, defendants concede a delegation of at least some seclusion decisions on a PRN basis to non-physician staff.

Nor is a witness’ competency to testify affected by the fact that he may be a disciple of schools, theories, and approaches to the diagnosis and treatment of mental disease that are different from those followed by the defendants. Such factors, however, may affect the weight of the testimony and whether, as a matter of discretion, the court will permit such testimony. A malpractice case is not made out because an expert disagrees with a defendant as to what is the best or better approach in treating a patient. Medicine is an inexact science and eminently qualified physicians may differ as to what constitutes a preferable course of treatment. Such differences as to preference do not amount to malpractice. To constitute evidence of malpractice, the proffered expert must testify clearly that there has been a departure from acceptable medical standards. Kiley v. Dervin, 314 Mass. 478, 50 N.E.2d 393 (1943); Bouffard v. Canby, 292 Mass. 305, 198 N.E. 253 (1935); Haggerty v. McCarthy, supra.

The expert witness’ education, training, experience and overall credentials, of course, affect the credibility of that opinion and the weight it will be given. That the admission of plaintiffs’ expert testimony here was properly within the court’s *1386discretion is underscored by the comments of Chief Justice Burger when a member of the D.C. Circuit:

[I]t would be proper, if a clinical psychologist is found qualified to testify as to the presence or absence of a mental disease and does so in opposition to a psychiatrist, to tell the jury they could take into account the difference in the education, training and experience of psychologists and psychiatrists and the absence of medical training in the former.

Jenkins v. United States, 113 U.S.App.D.C. 300, 314, 307 F.2d 637, 651 (D.C.Cir.1962) (concurring opinion).

In any event, that evidentiary issue is in a sense academic, as this court is not persuaded that any of the defendants was negligent in the treatment of any plaintiff.

1. Medication Negligence Claims

Although this court has already determined that an improper policy of forced medication was routinely followed at the Austin and May Units, it has also found that only Hunt, Bybel, Colleran, Wadsworth and Rogers of the named plaintiffs were forcibly medicated. Moreover, the fact that defendants’ forced medication was unconstitutional and warranted the granting of injunctive relief does not require a finding of negligence. That factor is one of many that may be considered in determining whether defendants’ treatment of the named plaintiffs fell below standards of reasonable medical care.

An additional factor to be considered is the medication itself, and whether its use was contrary to accepted medical standards. This court is persuaded by the weight of the evidence that it was reasonable medical practice for the defendants to utilize the psychotropic medication they did here. Regardless of possible adverse side effects, such medication is of significant value in treating psychotic patients. Plaintiffs have not persuaded this court that their use constituted medical malpractice.62

In addition, this court is not persuaded that defendants’ choices and prescribed dosages of psychotropic medication fell below acceptable medical standards. To be sure, those receiving the medication suffered predictable side effects. Indeed, it was necessary to alter Rogers’ treatment because of side effects.63 None of the named plaintiffs suffered from tardive dyskinesia or any permanent or prolonged disability as a result of the medication. To the contrary, most showed eventual improvement. In short, the fact that plaintiffs had a constitutional right to refuse psychotropic medication does not mean that its use was inconsistent with accepted medical standards.

An additional basis for plaintiffs’ medication malpractice claims is that the de*1387fendants failed in their duty to fully inform them as to the potential risks and benefits of their prescribed medication. Basically, that claim is grounded on the doctrine of “informed consent.”

That doctrine has yet to be formally recognized in Massachusetts as an independent tort cause of action. It is, nonetheless, a concept of “widespread recognition.” 64 Its theme is basically one of negligence — a failure on the part of a physician to provide sufficient information as to the competing risks and benefits of medication so that the patient can make an informed judgment whether or not to accept that treatment.65

The scope of a psychiatrist’s duty to inform mental health patients is not clearly defined. Defendants argue that the institutionalized patient lacks the capacity to make an informed decision on his treatment and that, in any event, disclosure of the potential side effects of medication may not be in the patient’s best interest.66 But this court has already rejected defendants’ notion of the presumed incompetence of committed patients to make treatment decisions. See Sec. VIII(A) supra. And, although disclosure of potential side effects of medication may be frightening to the mental patient, this court is not persuaded such a prospect, standing alone, justifies a failure to provide a patient with sufficient information to make an informed treatment decision.67 As with any other determination of reasonable conduct, whether a physician has sufficiently informed a patient depends on an analysis of all the surrounding circumstances.68

Even if the Supreme Judicial Court would recognize an “informed consent” cause of action, plaintiffs here would be unsuccessful in such a claim. First, this court is not persuaded that, during the relevant period of 1973 to 1975, a failure to inform a mental patient of a medication’s characteristics would have been considered sub-standard medical practice. And so, although the doctrine of informed consent has been implicitly accepted by this court in its injunctive decree, defendants may not be *1388held accountable in damages for failing in 1973 to anticipate the recognition in 1979 of a competent committed mental patient’s right to refuse treatment.

Moreover, the plaintiffs have not sustained their burden of proving that any of them has sustained the injury — tardive dyskinesia — that they claim the defendants should have warned them of prior to medicating. As the Supreme Judicial Court has remarked in dictum, “on any view of a doctrine of ‘informed consent,’ ” a plaintiff must prove not only that the undisclosed risk was material, but also “that it materialized.” Schroeder v. Lawrence, 372 Mass. 88, 359 N.E.2d 1301, 1303 (1977).

2. Seclusion Negligence Claims

This court has already determined that seclusion was routinely used as a non-emergency treatment modality at both the Austin and May Units, in violation of M.G.L.A. ch. 123, § 21. Plaintiffs Hunt, Bybel, Wadsworth, Warner and Bolden were secluded under non-emergency situations as part of behavior modification programs.

Defendants’ violation of M.G. L.A. ch. 123, § 21 does not require a finding of negligent medical practice. Violation of a “penal statute designed to secure safety” is not negligence per se when it does not appear “by express terms or by clear implication to have been the legislative intent.” Richmond v. Warren Institution for Savings, 307 Mass. 483, 485, 30 N.E.2d 407, 408 (1940).69

Although M.G.L.A. ch. 123, § 22 grants immunity to physicians who restrain patients in accordance with Section 21,70 that fact is not a clear expression of legislative intent to automatically impose liability for a deviation from the statutory mandate. Rather, defendants’ conduct must be measured by the standards and skill that prevailed in the years 1973 through 1975 among ordinary psychiatrists, Brune v. Belinkoff, supra, 354 Mass, at 109, and by the totality of relevant circumstances, including such factors as were considered by this court in allowing defendants’ good faith defense to plaintiffs’ § 1983 claims. See Sec. X(BX1) supra.

The court initially finds that, although the decision to seclude was usually made by the staff, the defendants were aware, or should have been, that seclusion was being routinely used as treatment, and that such a practice was contrary to law. The responsibility for seclusion decisions was the physicians’, not the staff’s. Defendants may not avoid that responsibility by arguing that they should not be vicariously liable for the acts of staff.

As noted, the plaintiffs were secluded in circumstances not constituting emergencies under Massachusetts law. There were two general categories of non-emergency seclusions: 1) seclusions in situations where the staff felt the patient was in need of restraint, even though the patient was not posing an imminent threat of extreme violence, personal injury or attempted suicide, and 2) seclusions as part of a treatment plan. With respect to the first category, this court does not find the defendants’ conduct to have been a departure from reasonable medical standards. Even in those instances when plaintiffs’ behavior precipitating seclusion constituted something less than physical emergency as defined by M.G.L.A. ch. 123, § 21, it was, nonetheless, highly disturbing to other patients and potentially disruptive. Although Massachusetts law proscribes seclusion in that situation, this court is not persuaded that defendants’ response to such disruptive behavior, under all the circumstances, was unreasonable.71 The weight of the evidence *1389persuades this court that defendants’ decision to seclude was a reasonable means of maintaining some stability and order on the ward, without causing disproportionate harm to the person secluded.72

With respect to the second category of seclusions, the court again finds no liability should attach to defendants’ conduct. Seclusion was used to remove the patient from the stimulus of the wards when it was considered necessary to do so in order to strengthen and maintain a patient’s sense of control. In the sense that it was intended to improve the patient’s ability to cope, seclusion in those instances was treatment. On the basis of the expert testimony and evidence presented at trial, this court is not persuaded that such treatment violated standards of reasonable medical practice73 even though proscribed by state statute.

The situation with respect to Donna Hunt, though more egregious, was nonetheless comparable. Seclusion was used to remove her from the stimulus of the wards and also as a means for modifying her behavior through negative reinforcement. As with all plaintiffs’ seclusion programs, there was no element of punishment, sadism or vindictiveness in Hunt’s treatment program. It was a studied effort to assist a most difficult patient in gaining control. As with the other named plaintiffs, this court is not persuaded that the defendants were negligent in formulating and implementing her seclusion programs.

On the basis of the evidence submitted, therefore, this court determines that plaintiffs have not met their burden of proving that defendants’ medication and seclusion practices failed to meet acceptable medical standards.74

10.3 Ford v. Wainwright 10.3 Ford v. Wainwright

FORD v. WAINWRIGHT, SECRETARY, FLORIDA DEPARTMENT OF CORRECTIONS

No. 85-5542.

Argued April 22, 1986

Decided June 26, 1986

*401MARSHALL, J., announced the judgment of the Court and delivered an opinion of the Court with respect to Parts I and II, in which BRENNAN, Blackmun, Powell, and Stevens, JJ., joined, and an opinion with respect to Parts III, IV, and V, in which Beennan, Blackmun, and Stevens, JJ., joined. Powell, J., filed an opinion concurring in part and concurring in the judgment, post, p. 418. O’Connor, J., filed an opinion concurring in the result in part and dissenting in part, in which White, J., joined, post, p. 427. Rehnquist, J., filed a dissenting opinion, in which BURGER, C. J., joined, post, p. 431.

Richard H. Burr III argued the cause for petitioner. With him on the briefs were Richard L. Jorandby, Craig S. Barnard, and Laurin A. Wollan, Jr.

Joy B. Shearer, Assistant Attorney General of Florida, argued the cause for respondent. With her on the brief was Jim Smith, Attorney General.*

Justice Marshall

announced the judgment of the Court and delivered the opinion of the Court with respect to Parts I and II and an opinion with respect to Parts III, IV, and V, in which Justice Brennan, Justice Blackmun, and Justice Stevens join.

For centuries no jurisdiction has countenanced the execution of the insane, yet this Court has never decided whether the Constitution forbids the practice. Today we keep faith with our common-law heritage in holding that it does.

H

Alvin Bernard Ford was convicted of murder in 1974 and sentenced to death. There is no suggestion that he was incompetent at the time of his offense, at trial, or at sentenc*402ing. In early 1982, however, Ford began to manifest gradual changes in behavior. They began as an occasional peculiar idea or confused perception, but became more serious over time. After reading in the newspaper that the Ku Klux Klan had held a rally in nearby Jacksonville, Florida, Ford developed an obsession focused upon the Klan. His letters to various people reveal endless brooding about his “Klan work,” and an increasingly pervasive delusion that he had become the target of a complex conspiracy, involving the Klan and assorted others, designed to force him to commit suicide. He believed that the prison guards, part of the conspiracy, had been killing people and putting the bodies in the concrete enclosures used for beds. Later, he began to believe that his women relatives were being tortured and sexually abused somewhere in the prison. This notion developed into a delusion that the people who were tormenting him at the prison had taken members of Ford’s family hostage. The hostage delusion took firm hold and expanded, until Ford was reporting that 135 of his friends and family were being held hostage in the prison, and that only he could help them. By “day 287” of the “hostage crisis,” the list of hostages had expanded to include “senators, Senator Kennedy, and many other leaders.” App. 53. In a letter to the Attorney General of Florida, written in 1983, Ford appeared to assume authority for ending the “crisis,” claiming to have fired a number of prison officials. He began to refer to himself as “Pope John Paul, III,” and reported having appointed nine new justices to the Florida Supreme Court. Id., at 59.

Counsel for Ford asked a psychiatrist who had examined Ford earlier, Dr. Jamal Amin, to continue seeing him and to recommend appropriate treatment. On the basis of roughly 14 months of evaluation, taped conversations between Ford and his attorneys, letters written by Ford, interviews with Ford’s acquaintances, and various medical records, Dr. Amin concluded in 1983 that Ford suffered from “a severe, uncontrollable, mental disease which closely resembles ‘Paranoid *403Schizophrenia With Suicide Potential’” — a “major mental disorder . . . severe enough to substantially affect Mr. Ford’s present ability to assist in the defense of his life.” Id., at 91.

Ford subsequently refused to see Dr. Amin again, believing him to have joined the conspiracy against him, and Ford’s counsel sought assistance from Dr. Harold Kaufman, who interviewed Ford in November 1983. Ford told Dr. Kaufman that “I know there is some sort of death penalty, but I’m free to go whenever I want because it would be illegal and the executioner would be executed.” Id., at 65. When asked if he would be executed, Ford replied: “I can’t be executed because of the landmark case. I won. Ford v. State will prevent executions all over.” Id., at 66. These statements appeared amidst long streams of seemingly unrelated thoughts in rapid succession. Dr. Kaufman concluded that Ford had no understanding of why he was being executed, made no connection between the homicide of which he had been convicted and the death penalty, and indeed sincerely believed that he would not be executed because he owned the prisons and could control the Governor through mind waves. Id., at 67. Dr. Kaufman found that there was “no reasonable possibility that Mr. Ford was dissembling, malingering or otherwise putting on a performance . . . .” Id., at 65. The following month, in an interview with his attorneys, Ford regressed further into nearly complete incomprehensibility, speaking only in a code characterized by intermittent use of the word “one,” making statements such as “Hands one, face one. Mafia one. God one, father one, Pope one. Pope one. Leader one.” Id., at 72.

Counsel for Ford invoked the procedures of Florida law governing the determination of competency of a condemned inmate, Fla. Stat. § 922.07 (1985). Following the procedures set forth in the statute, the Governor of Florida appointed a panel of three psychiatrists to evaluate whether, under §922.07(2), Ford had “the mental capacity to understand the nature of the death penalty and the reasons why it was im*404posed upon him.” At a single meeting, the three psychiatrists together interviewed Ford for approximately 30 minutes. Each doctor then filed a separate two- or three-page report with the Governor, to whom the statute delegates the final decision. One doctor concluded that Ford suffered from “psychosis with paranoia” but had “enough cognitive functioning to understand the nature and the effects of the death penalty, and why it is to be imposed on him.” App. 103. Another found that, although Ford was “psychotic,” he did “know fully what can happen to him.” Id., at 105-106. The third concluded that Ford had a “severe adaptational disorder,” but did “comprehend his total situation including being sentenced to death, and all of the implications of that penalty.” Id., at 99-100. He believed that Ford’s disorder, “although severe, seem[ed] contrived and recently learned.” Id., at 100. Thus, the interview produced three different diagnoses, but accord on the question of sanity as defined by state law.

The Governor’s decision was announced on April 30, 1984, when, without explanation or statement, he signed a death warrant for Ford’s execution. Ford’s attorneys unsuccessfully sought a hearing in state court to determine anew Ford’s competency to suffer execution. Ford v. Wainwright, 451 So. 2d 471, 475 (Fla. 1984). Counsel then filed a petition for habeas corpus in the United States District Court for the Southern District of Florida, seeking an evidentiary hearing on the question of Ford’s sanity, proffering the conflicting findings of the Governor-appointed commission and subsequent challenges to their methods by other psychiatrists. The District Court denied the petition without a hearing. The Court of Appeals granted a certificate of probable cause and stayed Ford’s execution, Ford v. Strickland, 734 F. 2d 538 (CA11 1984), and we rejected the State’s effort to vacate the stay of execution. Wainwright v. Ford, 467 U. S. 1220 (1984). The Court of Appeals then addressed the merits of Ford’s claim and a divided panel affirmed the Dis*405trict Court’s denial of the writ. 752 F. 2d 526 (CA11 1985). This Court granted Ford’s petition for certiorari in order to resolve the important issue whether the Eighth Amendment prohibits the execution of the insane and, if so, whether the District Court should have held a hearing on petitioner’s claim. 474 U. S. 1019 (1985).

I — I

Since this Court last had occasion to consider the infliction of the death penalty upon the insane, our interpretations of the Due Process Clause and the Eighth Amendment have evolved substantially. In Solesbee v. Balkcom, 339 U. S. 9 (1950), a condemned prisoner claimed a due process right to a judicial determination of his sanity, yet the Court did not consider the possible existence of a right under the Eighth Amendment, which had not yet been applied to the States. The sole question the Court addressed was whether Georgia’s procedure for ascertaining sanity adequately effectuated that State’s own policy of sparing the insane from execution. See also Caritativo v. California, 357 U. S. 549 (1958); United States ex rel. Smith v. Baldi, 344 U. S. 561 (1953); Phyle v. Duffy, 334 U. S. 431 (1948); Nobles v. Georgia, 168 U. S. 398 (1897). Now that the Eighth Amendment has been recognized to affect significantly both the procedural and the substantive aspects of the death penalty, the question of executing the insane takes on a wholly different complexion. The adequacy of the procedures chosen by a State to determine sanity, therefore, will depend upon an issue that this Court has never addressed: whether the Constitution places a substantive restriction on the State’s power to take the fife of an insane prisoner.

There is now little room for doubt that the Eighth Amendment’s ban on cruel and unusual punishment embraces, at a minimum, those modes or acts of punishment that had been considered cruel and unusual at the time that the Bill of Rights was adopted. See Solem v. Helm, 463 U. S. 277, 285-286 (1983); id., at 312-313 (Burger, C. J., joined by *406White, Rehnquist, and O’Connor, JJ., dissenting); Fur-man v. Georgia, 408 U. S. 238, 264 (1972) (Brennan, J., concurring); McGautha v. California, 402 U. S. 183, 226 (1971) (Black, J., concurring). “Although the Framers may have intended the Eighth Amendment to go beyond the scope of its English counterpart, their use of the language of the English Bill of Rights is convincing proof that they intended to provide at least the same protection . . . .” Solern v. Helm, supra, at 286.

Moreover, the Eighth Amendment’s proscriptions are not limited to those practices condemned by the common law in 1789. See Gregg v. Georgia, 428 U. S. 153, 171 (1976) (opinion of Stewart, Powell, and Stevens, JJ.). Not bound by the sparing humanitarian concessions of our forebears, the Amendment also recognizes the “evolving standards of decency that mark the progress of a maturing society.” Trap v. Dulles, 356 U. S. 86, 101 (1958) (plurality opinion). In addition to considering the barbarous methods generally outlawed in the 18th century, therefore, this Court takes into account objective evidence of contemporary values before determining whether a particular punishment comports with the fundamental human dignity that the Amendment protects. See Coker v. Georgia, 433 U. S. 584, 597 (1977) (plurality opinion).

A

We begin, then, with the common law. The bar against executing a prisoner who has lost his sanity bears impressive historical credentials; the practice consistently has been branded “savage and inhuman.” 4 W. Blackstone, Commentaries *24-*25 (hereinafter Blackstone). Blackstone explained:

“[IJdiots and lunatics are not chargeable for their own acts, if committed when under these incapacities: no, not even for treason itself. Also, if a man in his sound memory commits a capital offence, and before arraignment for it, he becomes mad, he ought not to be arraigned for *407it: because he is not able to plead to it with that advice and caution that he ought. And if, after he has pleaded, the prisoner becomes mad, he shall not be tried: for how can he make his defence? If, after he be tried and found guilty, he loses his senses before judgment, judgment shall not be pronounced; and if, after judgment, he becomes of nonsane memory, execution shall be stayed: for peradventure, says the humanity of the English law, had the prisoner been of sound memory, he might have alleged something in stay of judgment or execution.” Ibid, (footnotes omitted).

Sir Edward Coke had earlier expressed the same view of the common law of England: “[B]y intendment of Law the execution of the offender is for example, . . . but so it is not when a mad man is executed, but should be a miserable spectacle, both against Law, and of extream inhumanity and cruelty, and can be no example to others.” 3 E. Coke, Institutes 6 (6th ed. 1680) (hereinafter Coke). Other recorders of the common law concurred. See 1 M. Hale, Pleas of the Crown 36 (1736) (hereinafter Hale); 1 W. Hawkins, Pleas of the Crown 2 (7th ed. 1795) (hereinafter Hawkins); Hawles, Remarks on the Trial of Mr. Charles Bateman, 11 How. St. Tr. 474, 477 (1685) (hereinafter Hawles).

As is often true of common-law principles, see 0. Holmes, The Common Law 5 (1881), the reasons for the rule are less sure and less uniform than the rule itself. One explanation is that the execution of an insane person simply offends humanity, Coke 6; another, that it provides no example to others and thus contributes nothing to whatever deterrence value is intended to be served by capital punishment. Ibid. Other commentators postulate religious underpinnings: that it is uncharitable to dispatch an offender “into another world, when he is not of a capacity to fit himself for it,” Hawles 477. It is also said that execution serves no purpose in these cases because madness is its own punishment: furiosus *408solo furore punitur. Blackstone *395. More recent commentators opine that the community’s quest for “retribution” — the need to offset a criminal act by a punishment of equivalent “moral quality” — is not served by execution of an insane person, which has a “lesser value” than that of the crime for which he is to be punished. Hazard & Louisell, Death, the State, and the Insane: Stay of Execution, 9 UCLA L. Rev. 381, 387 (1962). Unanimity of rationale, therefore, we do not find. “But whatever the reason of the law is, it is plain the law is so.” Hawles 477. We know of virtually no authority condoning the execution of the insane at English common law.1

Further indications suggest that this solid proscription was carried to America, where it was early observed that “the judge is bound” to stay the execution upon insanity of the prisoner. 1 J. Chitty, A Practical Treatise on the Criminal Law *761; see 1 F. Wharton, A Treatise on Criminal Law §59 (8th ed. 1880).

B

This ancestral legacy has not outlived its time. Today, no State in the Union permits the execution of the insane.2 It *409is clear that the ancient and humane limitation upon the State’s ability to execute its sentences has as firm a hold upon the jurisprudence of today as it had centuries ago in England. The various reasons put forth in support of the common-law restriction have no less logical, moral, and practical force than they did when first voiced. For today, no less than before, we may seriously question the retributive value of executing a person who has no comprehension of why he has been singled out and stripped of his fundamental right to life. See Note, The Eighth Amendment and the Execution of the Presently Incompetent, 32 Stan. L. Rev. 765, 777, n. 58 (1980). Similarly, the natural abhorrence civilized societies feel at killing one who has no capacity to come to grips with his own conscience or deity is still vivid today. And the intuition that such an execution simply offends humanity is evidently shared across this Nation. Faced with such widespread evidence of a restriction upon sovereign power, this Court is compelled to conclude that the Eighth Amendment *410prohibits a State from carrying out a sentence of death upon a prisoner who is insane. Whether its aim be to protect the condemned from fear and pain without comfort of understanding, or to protect the dignity of society itself from the barbarity of exacting mindless vengeance, the restriction finds enforcement in the Eighth Amendment.

I — I 1 — I HH

The Eighth Amendment prohibits the State from inflicting the penalty of death upon a prisoner who is insane. Petitioner’s allegation of insanity in his habeas corpus petition, if proved, therefore, would bar his execution. The question before us is whether the District Court was under an obligation to hold an evidentiary hearing on the question of Ford’s sanity. In answering that question, we bear in mind that, while the underlying social values encompassed by the Eighth Amendment are rooted in historical traditions, the manner in which our judicial system protects those values is purely a matter of contemporary law. Once a substantive right or restriction is recognized in the Constitution, therefore, its enforcement is in no way confined to the rudimentary process deemed adequate in ages past.

A

In a habeas corpus proceeding, “a federal evidentiary hearing is required unless the state-court trier of fact has after a full hearing reliably found the relevant facts.” Townsend v. Sain, 372 U. S. 293, 312-313 (1963). The habeas corpus statute, following this Court’s decision in Townsend, provides that, in general, “a determination after a hearing on the merits of a factual issue, made by a State court of competent jurisdiction . . . , shall be presumed to be correct,” and an evidentiary hearing not required. 28 U. S. C. § 2254(d). In this case, it is clear that no state court has issued any determination to which that presumption of correctness could be said to attach; indeed, no court played any role in the rejection of petitioner’s claim of insanity. Thus, quite simply, *411Townsend and §2254 require the District Court to grant a hearing de novo on that question.

But our examination does not stop there. For even when a state court has rendered judgment, a federal court is obliged to hold an evidentiary hearing on habeas corpus if, among other factors, “the factfinding procedure employed by the State court was not adequate to afford a full and fair hearing,” § 2254(d)(2); or “the material facts were not adequately developed at the State court hearing,” § 2254(d)(3); or “the applicant did not receive a full, fair, and adequate hearing in the State court proceeding.” § 2254(d)(6). If federal factfinding is to be avoided, then, in addition to providing a court judgment on the constitutional question, the State must also ensure that its procedures are adequate for the purpose of finding the facts.

B

The adequacy of a state-court procedure under Townsend is largely a function of the circumstances and the interests at stake. In capital proceedings generally, this Court has demanded that factfinding procedures aspire to a heightened standard of reliability. See, e. g., Spaziano v. Florida, 468 U. S. 447, 456 (1984). This especial concern is a natural consequence of the knowledge that execution is the most irremediable and unfathomable of penalties; that death is different. See Woodson v. North Carolina, 428 U. S. 280, 305 (1976) (opinion of Stewart, Powell, and Stevens, JJ.).

Although the condemned prisoner does not enjoy the same presumptions accorded a defendant who has yet to be convicted or sentenced, he has not lost the protection of the Constitution altogether; if the Constitution renders the fact or timing of his execution contingent upon establishment of a further fact, then that fact must be determined with the high regard for truth that befits a decision affecting the life or death of a human being. Thus, the ascertainment of a prisoner’s sanity as a predicate to lawful execution calls for no less stringent standards than those demanded in any *412other aspect of a capital proceeding. Indeed, a particularly acute need for guarding against error inheres in a determination that “in the present state of the mental sciences is at best a hazardous guess however conscientious.” Solesbee v. Balkcom, 339 U. S., at 23 (Frankfurter, J., dissenting). That need is greater still because the ultimate decision will turn on the finding of a single fact, not on a range of equitable considerations. Cf. Woodson v. North Carolina, supra, at 304. In light of these concerns, the procedures employed in petitioner’s case do not fare well.

C

Florida law directs the Governor, when informed that a person under sentence of death may be insane, to stay the execution and appoint a commission of three psychiatrists to examine the prisoner. Fla. Stat. §922.07 (1985 and Supp. 1986). “The examination of the convicted person shall take place with all three psychiatrists present at the same time.” Ibid. After receiving the report of the commission, the Governor must determine whether “the convicted person has the mental capacity to understand the nature of the death penalty and the reasons why it was imposed on him.” Ibid. If the Governor finds that the prisoner has that capacity, then a death warrant is issued; if not, then the prisoner is committed to a mental health facility. The procedure is conducted wholly within the executive branch, ex parte, and provides the exclusive means for determining sanity. Ford v. Wainwright, 451 So. 2d, at 475.

Petitioner received the statutory process. The Governor selected three psychiatrists, who together interviewed Ford for a total of 30 minutes, in the presence of eight other people, including Ford’s counsel, the State’s attorneys, and correctional officials. The Governor’s order specifically directed that the attorneys should not participate in the examination in any adversarial manner. This order was consistent with the present Governor’s “publicly announced pol*413icy of excluding all advocacy on the part of the condemned from the process of determining whether a person under a sentence of death is insane.” Goode v. Wainwright, 448 So. 2d 999, 1001 (Fla. 1984).

After submission of the reports of the three examining psychiatrists, reaching conflicting diagnoses but agreeing on the ultimate issue of competency, Ford’s counsel attempted to submit to the Governor some other written materials, including the reports of the two other psychiatrists who had examined Ford at greater length, one of whom had concluded that the prisoner was not competent to suffer execution. The Governor’s office refused to inform counsel whether the submission would be considered. The Governor subsequently issued his decision in the form of a death warrant. That this most cursory form of procedural review fails to achieve even the minimal degree of reliability required for the protection of any constitutional interest, and thus falls short of adequacy under Townsend, is self-evident.

IV

A

The first deficiency in Florida’s procedure lies in its failure to include the prisoner in the truth-seeking process. Notwithstanding this Court’s longstanding pronouncement that “[t]he fundamental requisite of due process of law is the opportunity to be heard,” Grannis v. Órdean, 234 U. S. 385, 394 (1914), state practice does not permit any material relevant to the ultimate decision to be submitted on behalf of the prisoner facing execution. In all other proceedings leading to the execution of an accused, we have said that the factfinder must “have before it all possible relevant information about the individual defendant whose fate it must determine.” Jurek v. Texas, 428 U. S. 262, 276 (1976) (plurality opinion). And we have forbidden States to limit the capital defendant’s submission of relevant evidence in mitigation of the sentence. Skipper v. South Carolina, 476 U. S. 1, 8 *414(1986); Lockett v. Ohio, 438 U. S. 586, 604 (1978) (joint opinion). It would be odd were we now to abandon our insistence upon unfettered presentation of relevant information, before the final fact antecedent to execution has been found.

Rather, consistent with the heightened concern for fairness and accuracy that has characterized our review of the process requisite to the taking of a human life, we believe that any procedure that precludes the prisoner or his counsel from presenting material relevant to his sanity or bars consideration of that material by the factfinder is necessarily inadequate. “[T]he minimum assurance that the life-and-death guess will be a truly informed guess requires respect for the basic ingredient of due process, namely, an opportunity to be allowed to substantiate a claim before it is rejected.” Solesbee v. Balkcom, supra, at 23 (Frankfurter, J., dissenting).

We recently had occasion to underscore the value to be derived from a factfinder’s consideration of differing psychiatric opinions when resolving contested issues of mental state. In Ake v. Oklahoma, 470 U. S. 68 (1985), we recognized that, because “psychiatrists disagree widely and frequently on what constitutes mental illness [and] on the appropriate diagnosis to be attached to given behavior and symptoms,” the factfinder must resolve differences in opinion within the psychiatric profession “on the basis of the evidence offered by each party” when a defendant’s sanity is at issue in a criminal trial. Id., at 81. The same holds true after conviction; without any adversarial assistance from the prisoner’s representative — especially when the psychiatric opinion he proffers is based on much more extensive evaluation than that of the state-appointed commission — the factfinder loses the substantial benefit of potentially probative information. The result is a much greater likelihood of an erroneous decision.

*415B

A related flaw in the Florida procedure is the denial of any opportunity to challenge or impeach the state-appointed psychiatrists’ opinions. “[C]ross-examination ... is beyond any doubt the greatest legal engine ever invented for the discovery of truth.” 5 J. Wigmore, Evidence § 1367 (J. Chadbourn rev. 1974). Cross-examination of the psychiatrists, or perhaps a less formal equivalent, would contribute markedly to the process of seeking truth in sanity disputes by bringing to light the bases for each expert’s beliefs, the precise factors underlying those beliefs, any history of error or caprice of the examiner, any personal bias with respect to the issue of capital punishment, the expert’s degree of certainty about his or her own conclusions, and the precise meaning of ambiguous words used in the report. Without some questioning of the experts concerning their technical conclusions, a factfinder simply cannot be expected to evaluate the various opinions, particularly when they are themselves inconsistent. See Barefoot v. Estelle, 463 U. S. 880, 899 (1983). The failure of the Florida procedure to afford the prisoner’s representative any opportunity to clarify or challenge the state experts’ opinions or methods creates a significant possibility that the ultimate decision made in reliance on those experts will be distorted.3

*416c

Perhaps the most striking defect in the procedures of Fla. Stat. §922.07 (1985 and Supp. 1986), as noted earlier, is the State’s placement of the decision wholly within the executive branch. Under this procedure, the person who appoints the experts and ultimately decides whether the State will be able to carry out the sentence that it has long sought is the Governor, whose subordinates have been responsible for initiating every stage of the prosecution of the condemned from arrest through sentencing. The commander of the State’s corps of prosecutors cannot be said to have the neutrality that is necessary for reliability in the factfinding proceeding.

Historically, delay of execution on account of insanity was not a matter of executive clemency (ex mandato regis) or judicial discretion (ex arbitrio judiéis); rather, it was required by law (ex necessitate legis). 1 N. Walker, Crime and Insanity in England 196 (1968). Thus, history affords no better basis than does logic for placing the final determination of a fact, critical to the trigger of a constitutional limitation upon the State’s power, in the hands of the State’s own chief executive. In no other circumstance of which we are aware is the vindication of a constitutional right entrusted to the unreviewable discretion of an administrative tribunal.

V

A

Having identified various failings of the Florida scheme, we must conclude that the State’s procedures for determining sanity are inadequate to preclude federal redetermination of the constitutional issue. We do not here suggest that only a full trial on the issue of sanity will suffice to protect the federal interests; we leave to the State the task of developing appropriate ways to enforce the constitutional restriction *417upon its execution of sentences.4 It may be that some high threshold showing on behalf of the prisoner will be found a necessary means to control the number of nonmeritorious or repetitive claims of insanity. Cf. Pate v. Robinson, 383 U. S. 375, 387 (1966) (hearing on competency to stand trial required if “sufficient doubt” of competency exists). Other legitimate pragmatic considerations may also supply the boundaries of the procedural safeguards that feasibly can be provided.

Yet the lodestar of any effort to devise a procedure must be the overriding dual imperative of providing redress for those with substantial claims and of encouraging accuracy in the factfinding determination. The stakes are high, and the “evidence” will always be imprecise. It is all the more important that the adversary presentation of relevant information be as unrestricted as possible. Also essential is that the manner of selecting and using the experts responsible for producing that “evidence” be conducive to the formation of neutral, sound, and professional judgments as to the prisoner’s ability to comprehend the nature of the penalty. Fidelity to these principles is the solemn obligation of a civilized society.

B

Today we have explicitly recognized in our law a principle that has long resided there. It is no less abhorrent today than it has been for centuries to exact in penance the life of one whose mental illness prevents him from comprehending the reasons for the penalty or its implications. In fight of the *418clear need for trustworthiness in any factual finding that will prevent or permit the carrying out of an execution, we hold that Fla. Stat. §922.07 (1985 and Supp. 1986) provides inadequate assurances of accuracy to satisfy the requirements of Townsend v. Sain, 372 U. S. 293 (1963). Having been denied a factfinding procedure “adequate to afford a full and fair hearing” on the critical issue, 28 U. S. C. § 2254(d)(2), petitioner is entitled to an evidentiary hearing in the District Court, de novo, on the question of his competence to be executed. Townsend v. Sain, swpra, at 312.

The judgment of the Court of Appeals is reversed, and the case is remanded for further proceedings consistent with this opinion.

It is so ordered.

Justice Powell,

concurring in part and concurring in the judgment.

I join Parts I and II of the Court’s opinion. As Justice Marshall ably demonstrates, execution of the insane was barred at common law precisely because it was considered cruel and unusual. In Solem v. Helm, 463 U. S. 277 (1983), we explained that while the Framers “may have intended the Eighth Amendment to go beyond the scope of its English counterpart, their use of the language of the English Bill of Rights is convincing proof that they intended to provide at least the same protection.” Id., at 286. It follows that the practice of executing the insane is barred by our own Constitution.

That conclusion leaves two issues for our determination: (i) the meaning of insanity in this context, and (ii) the procedures States must follow in order to avoid the necessity of de novo review in federal courts under 28 U. S. C. § 2254(d). The Court’s opinion does not address the first of these issues, and as to the second, my views differ substantially from Justice Marshall’s. I therefore write separately.

*419I

The Court holds today that the Eighth Amendment bars execution of a category of defendants defined by their mental state. The bounds of that category are necessarily governed by federal constitutional law. I therefore turn to the same sources that give rise to the substantive right to determine its precise definition: chiefly, our common-law heritage and the modern practices of the States, which are indicative of our “evolving standards of decency.” Trop v. Dulles, 356 U. S. 86, 101 (1958) (plurality opinion). See Solem v. Helm, supra, at 284-286; Gregg v. Georgia, 428 U. S. 153, 175-176 (1976) (opinion of Stewart, Powell, and Stevens, JJ.).

A

As the Court recognizes, ante, at 407-408, the ancient prohibition on execution of the insane rested on differing theories. Those theories do not provide a common answer when it comes to defining the mental awareness required by the Eighth Amendment as a prerequisite to a defendant’s execution. On the one hand, some authorities contended that the prohibition against executing the insane was justified as a way of preserving the defendant’s ability to make arguments on his own behalf. See 1 M. Hale, Pleas of the Crown 35 (1736) (“if after judgment he become of non sane memory, his execution shall be spared; for were he of sound memory he might allege somewhat in stay of judgment or execution”); accord 4 W. Blackstone, Commentaries *388-*389. Other authorities suggest, however, that the prohibition derives from more straightforward humanitarian concerns. Coke expressed the view that execution was intended to be an “example” to the living, but that the execution of “a mad man” was such “a miserable spectacle ... of extream inhumanity and cruelty” that it “can be no example to others.” 3 E. Coke, Institutes 6 (1794). Hawles added that it is “against Christian charity to send a great offender quick . . . into another world, when he is not of a capacity to fit himself for it.” *420Hawles, Remarks on the Trial of Mr. Charles Bateman, 11 How. St. Tr. 474, 477 (1685).

The first of these justifications has slight merit today. Modern practice provides far more extensive review of convictions and sentences than did the common law, including not only direct appeal but ordinarily both state and federal collateral review.1 Throughout this process, the defendant has access to counsel, by constitutional right at trial, and by employment or appointment at other stages of the process whenever the defendant raises substantial claims. Nor does the defendant merely have the right to counsel’s assistance; he also has the right to the effective assistance of counsel at trial and on appeal. Evitts v. Lucey, 469 U. S. 387 (1985); Strickland v. Washington, 466 U. S. 668 (1984). See Kimmelman v. Morrison, ante, at 392-393 (Powell, J., concurring in judgment). These guarantees are far broader than those enjoyed by criminal defendants at common law. It is thus unlikely indeed that a defendant today could go to his death with knowledge of undiscovered trial error that might set him free.

In addition, in cases tried at common law execution often followed fairly quickly after trial, so that incompetence at the *421time of execution was linked as a practical matter with incompetence at the trial itself. Our decisions already recognize, however, that a defendant must be competent to stand trial, and thus the notion that a defendant must be able to assist in his defense is largely provided for. See Drope v. Missouri, 420 U. S. 162 (1975).2

B

The more general concern of the common law — that executions of the insane are simply cruel — retains its vitality. It is as true today as when Coke lived that most men and women value the opportunity to prepare, mentally and spiritually, for their death. Moreover, today as at common law, one of the death penalty’s critical justifications, its retributive force, depends on the defendant’s awareness of the penalty’s existence and purpose. Thus, it remains true that executions of the insane both impose a uniquely cruel penalty and are inconsistent with one of the chief purposes of executions generally. For precisely these reasons, Florida requires the Governor to stay executions of those who “d[o] not have the mental capacity to understand the nature of the death penalty and why it was imposed” on them. Fla. Stat. §922.07 (1985 and Supp. 1986). See also Ill. Rev. Stat., ch. 38, ¶ 1005-2-3(a) (1985) (“A person is unfit to be executed if because of a mental condition he is unable to understand the nature and purpose of such sentence”); State v. Pastet, 169 Conn. 13, 28, 363 A. 2d 41, 49 (question is “whether the defendant was able to understand the nature of the sentencing proceedings, i. e., why he was being punished and the nature of his punishment”), cert, denied, 423 U. S. 937 (1975). A number of *422States have more rigorous standards,3 but none disputes the need to require that those who are executed know the fact of their impending execution and the reason for it.

Such a standard appropriately defines the kind of mental deficiency that should trigger the Eighth Amendment prohibition. If the defendant perceives the connection between his crime and his punishment, the retributive goal of the criminal law is satisfied. And only if the defendant is aware that his death is approaching can he prepare himself for his passing. Accordingly, I would hold that the Eighth Amendment forbids the execution only of those who are unaware of the punishment they are about to suffer and why they are to suffer it.

Petitioner’s claim of insanity plainly fits within this standard. According to petitioner’s proffered psychiatric examination, petitioner does not know that he is to be executed, but rather believes that the death penalty has been invalidated. App. 65-67. If this assessment is correct, petitioner *423cannot connect his execution to the crime for which he was convicted. Thus, the question is whether petitioner’s evidence entitles him to a hearing in Federal District Court on his claim.

II

Petitioner concedes that the Governor of Florida has determined that he is not insane under the standard prescribed by Florida’s statute, which is the same as the standard just described. Petitioner further concedes that there is expert evidence that supports the Governor’s finding. Thus, if that finding is entitled to a presumption of correctness under 28 U. S. C. § 2254(d), there is no ground for holding a hearing on petitioner’s federal habeas corpus petition.

I agree with Justice Marshall that the Governor’s finding is not entitled to a presumption of correctness under § 2254(d). I reach this conclusion for two independent reasons. First, § 2254(d) requires deference to the factual findings of “a State court of competent jurisdiction.” The term “State court” may have a certain amount of flexibility,4 but no amount of stretching can extend it to include the Governor. The essence of a “court” is independence from the prosecutorial arm of government and, as Justice Marshall correctly notes, the Governor is “[t]he commander of the State’s corps of prosecutors.” Ante, at 416. Unless the relevant language is to be read out of the statute, I see no basis for affording any deference to the Governor’s determination.

Second, the presumption of correctness does not attach to the Governor’s implicit finding of sanity because the State did not give petitioner’s claim “a full and fair hearing,” 28 U. S. C. § 2254(d)(2). This statutory phrase apparently was drawn from the Court’s opinion in Townsend v. Sain, 372 U. S. 293, 313 (1963). There, the Court concluded that where the state court’s “fact-finding procedure . . . was not *424adequate for reaching reasonably correct results,” or where the process “appeared] to be seriously inadequate for the ascertainment of the truth,” no presumption of correctness would attach to the state court’s findings when those findings were challenged on federal habeas corpus. Id., at 316.

At least in the context of competency determinations prior to execution, this standard is no different from the protection afforded by procedural due process. It is clear that an insane defendant’s Eighth Amendment interest in forestalling his execution unless or until he recovers his sanity cannot be deprived without a “fair hearing.” Indeed, fundamental fairness is the hallmark of the procedural protections afforded by the Due Process Clause. See Lassiter v. Department of Social Services of Durham County, 452 U. S. 18, 24-25 (1981). Thus, the question in this case is whether Florida’s procedures for determining petitioner’s sanity comport with the requirements of due process.

Together with Justice Marshall and Justice O’Con-nor, I would hold that they do not. As Justice O’Connor states, “[i]f there is one ‘fundamental requisite’ of due process, it is that an individual is entitled to an ‘opportunity to be heard.’” Post, at 430 (quoting Grannis v. Ordean, 234 U. S. 385, 394 (1914)). In this case, petitioner was deprived of that opportunity. The Florida statute does not require the Governor to consider materials submitted by the prisoner, and the present Governor has a “publicly announced policy of excluding” such materials from his consideration. Goode v. Wainwright, 448 So. 2d 999, 1001 (Fla. 1984). Thus, the determination of petitioner’s sanity appears to have been made solely on the basis of the examinations performed by state-appointed psychiatrists. Such a procedure invites arbitrariness and error by preventing the affected parties from offering contrary medical evidence or even from explaining the inadequacies of the State’s examinations. It does not, therefore, comport with due process. It follows that the State’s procedure was not “fair,” and that the Dis*425trict Court on remand must consider the question of petitioner’s competency to be executed.

I — H I — I I — I

While the procedures followed by Florida in this case do not comport with basic fairness, I would not require the kind of full-scale “sanity trial” that Justice Marshall appears to find necessary. See ante, at 413-416, 418. Due process is a flexible concept, requiring only “such procedural protections as the particular situation demands.” Mathews v. Eldridge, 424 U. S. 319, 334 (1976); Morrissey v. Brewer, 408 U. S. 471, 481 (1972). See also post, at 429 (O’Connor, J., concurring in result in part and dissenting in part). In this instance, a number of considerations support the conclusion that the requirements of due process are not as elaborate as Justice Marshall suggests.

First, the Eighth Amendment claim at issue can arise only after the prisoner has been validly convicted of a capital crime and sentenced to death. Thus, in this case the State has a substantial and legitimate interest in taking petitioner’s life as punishment for his crime. That interest is not called into question by petitioner’s claim. Rather, the only question raised is not whether, but when, his execution may take place.5 This question is important, but it is not comparable to the antecedent question whether petitioner should be executed at all. It follows that this Court’s decisions imposing heightened procedural requirements on capital trials and sentencing proceedings — e. g., Lockett v. Ohio, 438 U. S. 586 (1978) (plurality opinion); Turner v. Murray, 476 U. S. 28 (1986) — do not apply in this context.

Second, petitioner does not make his claim of insanity against a neutral background. On the contrary, in order to *426have been convicted and sentenced, petitioner must have been judged competent to stand trial, or his competency-must have been sufficiently clear as not to raise a serious question for the trial court. The State therefore may properly presume that petitioner remains sane at the time sentence is to be carried out,6 and may require a substantial threshold showing of insanity merely to trigger the hearing process. Cf. Ake v. Oklahoma, 470 U. S. 68, 82-83 (1985).

Finally, the sanity issue in this type of case does not resemble the basic issues at trial or sentencing. Unlike issues of historical fact, the question of petitioner’s sanity calls for a basically subjective judgment. See Addington v. Texas, 441 U. S. 418, 429-430 (1979); cf. Barefoot v. Estelle, 463 U. S. 880, 898-901 (1983). And unlike the determination of whether the death penalty is appropriate in a particular case, the competency determination depends substantially on expert analysis in a discipline fraught with “subtleties and nuances.” Addington, supra, at 430. This combination of factors means that ordinary adversarial procedures — complete with live testimony, cross-examination, and oral argument by counsel — are not necessarily the best means of arriving at sound, consistent judgments as to a defendant’s sanity. Cf. Parham v. J. R., 442 U. S. 584, 609 (1979) (“Common human experience and scholarly opinions suggest that the supposed protections of an adversary proceeding to determine the appropriateness of medical decisions for the commitment and treatment of mental and emotional illness may well be more illusory than real”).

*427We need not determine the precise limits that due process imposes in this area. In general, however, my view is that a constitutionally acceptable procedure may be far less formal than a trial. The State should provide an impartial officer or board that can receive evidence and argument from the prisoner’s counsel, including expert psychiatric evidence that may differ from the State’s own psychiatric examination. Beyond these basic requirements, the States should have substantial leeway to determine what process best balances the various interests at stake. As long as basic fairness is observed, I would find due process satisfied, and would apply the presumption of correctness of § 2254(d) on federal habeas corpus.

> HH

Because petitioner has raised a viable claim under the Eighth Amendment, and because that claim was not adjudicated fairly within the meaning of due process or of § 2254(d), petitioner is entitled to have his claim adjudicated by the District Court on federal habeas corpus. I therefore join the Court’s judgment.

Justice O’Connor,

with whom Justice White joins, concurring in the result in part and dissenting in part.

I am in full agreement with Justice Rehnquist’s conclusion that the Eighth Amendment does not create a substantive right not to be executed while insane. Accordingly, I do not join the Court’s reasoning or opinion. Because, however, the conclusion is for me inescapable that Florida positive law has created a protected liberty interest in avoiding execution while incompetent, and because Florida does not provide even those minimal procedural protections required by due process in this area, I would vacate the judgment and remand to the Court of Appeals with directions that the case be returned to the Florida system so that a hearing can be held in a manner consistent with the requirements of the Due Process Clause. I cannot agree, however, that the federal *428courts should have any role whatever in the substantive determination of a defendant’s competency to be executed.

As we explained in Hewitt v. Helms, 459 U. S. 460, 466 (1983), “[l]iberty interests protected by the Fourteenth Amendment may arise from two sources — the Due Process Clause itself and the laws of the States.” See also Meachum v. Fano, 427 U. S. 215, 223-227 (1976). With Justice Rehnquist, I agree that the Due Process Clause does not independently create a protected interest in avoiding the execution of a death sentence during incompetency. See also Solesbee v. Balkcom, 339 U. S. 9 (1950). The relevant provision of the Florida statute, however, provides that the Governor “shall” have the prisoner committed to a “Department of Corrections mental health treatment facility” if the prisoner “does not have the mental capacity to understand the nature of the death penalty and why it was imposed on him.” Fla. Stat. § 922.07(3) (1985 and Supp. 1986). Our cases leave no doubt that where a statute indicates with “language of an unmistakable mandatory character,” that state conduct injurious to an individual will not occur “absent specified substantive predicates,” the statute creates an expectation protected by the Due Process Clause. Hewitt v. Helms, swpra, at 471-472. See also Vitek v. Jones, 445 U. S. 480, 488-491 (1980); Greenholtz v. Nebraska Penal Inmates, 442 U. S. 1, 10 (1979) (entitlement created where under state law “there is [a] set of facts which, if shown, mandate a decision favorable to the individual”). That test is easily met here. Nor is it relevant that the statute creating the interest also specifies the procedures to be followed when the State seeks to deprive the individual of that interest. As we reaffirmed last Term, “[t]he categories of substance and procedure are distinct.” Cleveland Board of Education v. Loudermill, 470 U. S. 532, 541 (1985). Thus, regardless of the procedures the State deems adequate for determining the preconditions to adverse official action, federal law defines the kind of proc*429ess a State must afford prior to depriving an individual of a protected liberty or property interest. Id., at 541.

Although the state-created entitlement to avoid execution while insane unquestionably triggers the demands of the Due Process Clause, in my judgment those demands are minimal in this context. “It is axiomatic that due process ‘is flexible and calls for such procedural protections as the particular situation [requires].’” Greenholtz v. Nebraska Penal Inmates, supra, at 12, quoting Morrissey v. Brewer, 408 U. S. 471, 481 (1972). And there are any number of reasons for concluding that this “particular situation” warrants substantial caution before reading the Due Process Clause to mandate anything like the full panoply of trial-type procedures. The prisoner’s interest in avoiding an erroneous determination is, of course, very great. But I consider it self-evident that once society has validly convicted an individual of a crime and therefore established its right to punish, the demands of due process are reduced accordingly. Meachum v. Fano, supra, at 224. Moreover, the potential for false claims and deliberate delay in this context is obviously enormous. Nobles v. Georgia, 168 U. S. 398, 405-406 (1897). This potential is exacerbated by a unique feature of the prisoner’s protected interest in suspending the execution of a death sentence during incompetency. By definition, this interest can never be conclusively and finally determined: Regardless of the number of prior adjudications of the issue, until the very moment of execution the prisoner can claim that he has become insane sometime after the previous determination to the contrary. Hazard & Louisell, Death, the State and the Insane: Stay of Execution, 9 UCLA L. Rev. 381, 399-400 (1962). These difficulties, together with the fact that the issue arises only after conviction and sentencing, convince me that the Due Process Clause imposes few requirements on the States in this context.

Even given the broad latitude I would leave to the States in this area, however, I believe that one aspect of the Florida *430procedure for determining competency to be executed renders that procedure constitutionally deficient. If there is one “fundamental requisite” of due process, it is that an individual is entitled to an “opportunity to be heard.” Grannis v. Ordean, 234 U. S. 385, 394 (1914). As currently implemented, the Florida procedure for determining competency violates this bedrock principle. By Executive Order, the present Governor has provided that “[c]ounsel for the inmate and the State Attorney may be present [at the competency hearing] but shall not participate in the examination in any adversarial manner.” Exec. Order No. 83-137 (Dec. 9, 1983). See also Goode v. Wainwright, 448 So. 2d 999, 1001 (Fla. 1984) (describing the Governor’s “publicly announced policy of excluding all advocacy on the part of the condemned from the process of determining whether a person under a sentence of death is insane”). Indeed, respondent does not dispute that the Governor’s office has steadfastly refused to acknowledge whether it would even review the extensive psychiatric materials submitted by petitioner concerning his present mental state. While I would not invariably require oral advocacy or even cross-examination, due process at the very least requires that the decisionmaker consider the prisoner’s written submissions.

I conclude therefore that Florida law has created a protected expectation that no execution will be carried out while the prisoner lacks the “mental capacity to understand the nature of the death penalty and why it was imposed on him.” Fla. Stat. §922.07(3) (1985). Because Florida’s procedures are inadequate to satisfy even the minimal requirements of due process in this context, I would vacate the judgment below with instructions that the case be returned to Florida so that it might assess petitioner’s competency in a manner that accords with the command of the Fourteenth Amendment. In my view, however, the only federal question presented in cases such as this is whether the State’s positive law has created a liberty interest and whether its procedures *431are adequate to protect that interest from arbitrary deprivation. Once satisfied that the procedures were adequate, a federal court has no authority to second-guess a State’s substantive competency determination.

Justice Rehnquist,

with whom The Chief Justice joins, dissenting.

The Court today holds that the Eighth Amendment prohibits a State from carrying out a lawfully imposed sentence of death upon a person who is currently insane. This holding is based almost entirely on two unremarkable observations. First, the Court states that it “know[s] of virtually no authority condoning the execution of the insane at English common law.” Ante, at 408. Second, it notes that “[t]oday, no State in the Union permits the execution of the insane.” Ibid. Armed with these facts, and shielded by the claim that it is simply “keep[ing] faith with our common-law heritage,” ante, at 401, the Court proceeds to cast aside settled precedent and to significantly alter both the common-law and current practice of not executing the insane. It manages this feat by carefully ignoring the fact that the Florida scheme it finds unconstitutional, in which the Governor is assigned the ultimate responsibility of deciding whether a condemned prisoner is currently insane, is fully consistent with the “common-law heritage” and current practice on which the Court purports to rely.

The Court places great weight on the “impressive historical credentials” of the common-law bar against executing a prisoner who has lost his sanity. Ante, at 406-408. What it fails to mention, however, is the equally important and unchallenged fact that at common law it was the executive who passed upon the sanity of the condemned. See 1 N. Walker, Crime and Insanity in England 194-203 (1968). So when the Court today creates a constitutional right to a determination of sanity outside of the executive branch, it does so not in keeping with but at the expense of “our common-law heritage.”

*432In Solesbee v. Balkcom, 339 U. S. 9 (1950), a condemned prisoner claimed that he had a constitutional right to a judicial determination of his sanity. There, as here, the State did not approve the execution of insane persons and vested in the Governor the responsibility for determining, with the aid of experts, the sanity vel non of persons sentenced to death. In rejecting the prisoner’s claim, this Court stated:

“Postponement of execution because of insanity bears a close affinity not to trial for a crime but rather to reprieves of sentences in general. The power to reprieve has usually sprung from the same source as the power to pardon. Power of executive clemency in this country undoubtedly derived from the practice as it had existed in England. Such power has traditionally rested in governors or the President, although some of that power is often delegated to agencies such as pardon or parole boards. Seldom, if ever, has this power of executive clemency been subjected to review by the courts.” Id., at 11-12.

Despite references to “evolving standards of decency,” ante, at 406, and “the jurisprudence of today,” ante, at 409, the Court points to no change since Solesbee in the States’ approach to determining the sanity of a condemned prisoner. Current statutes quite often provide that initiation of inquiry into and/or final determination of postsentencing insanity is a matter for the executive or the prisoner’s custodian.* The Court’s profession of “faith to our common-law heritage” and *433“evolving standards of decency” is thus at best a half-truth. It is Florida’s scheme — which combines a prohibition against execution of the insane with executive-branch procedures for evaluating claims of insanity — that is more faithful to both traditional and modern practice. And no matter how longstanding and universal, laws providing that the State should not execute persons the executive finds insane are not themselves sufficient to create an Eighth Amendment right that sweeps away as inadequate the procedures for determining sanity crafted by those very laws.

Petitioner makes the alternative argument, not reached by the Court, that even if the Eighth Amendment does not prohibit execution of the insane, Florida’s decision to bar such executions creates a right in condemned persons to trial-type procedures to determine sanity. Here, too, Solesbee is instructive:

“Recently we have pointed out the necessary and inherent differences between trial procedures and post-conviction procedures such as sentencing. Williams v. New York, 337 U. S. 241. In that case we emphasized that certain trial procedure safeguards are not applicable to the process of sentencing. This principle applies even more forcefully to an effort to transplant every trial safeguard to a determination of sanity after conviction. As was pointed out in [Nobles v. Georgia, 168 U. S. 398 (1897)], to require judicial review every time a convicted defendant suggested insanity would make the possibility of carrying out a sentence depend upon ‘fecundity in making suggestion after suggestion of insanity.’ Nobles v. Georgia, supra, at 405-406. See also Phyle v. Duffy, [334 U. S. 431 (1948)]. To protect itself society must have power to try, convict, and execute sentences. Our legal system demands that this governmental duty be performed with scrupulous fairness to the accused. We cannot say that it offends due process to leave the question of a convicted person’s sanity to the solemn respon*434sibility of a state’s highest executive with authority to invoke the aid of the most skillful class of experts on the crucial questions involved.” 339 U. S., at 12-13.

Even the sole dissenter in Solesbee, Justice Frankfurter, agreed that if the Constitution afforded condemned prisoners no substantive right not to be executed when insane, then the State would be free to place on the Governor the responsibility for determining sanity. Id., at 15.

Petitioner argues that Solesbee is no longer controlling because it was decided “at a time when due process analysis still turned on the right-privilege distinction.” Brief for Petitioner 8. But as petitioner concedes, his due process claim turns on a showing that the Florida statute at issue here created an individual right not to be executed while insane. Even a cursory reading of the statute reveals that the only right it creates in a condemned prisoner is to inform the Governor that the prisoner may be insane. Fla. Stat. § 922.07(1) (1985). The only legitimate expectation it creates is that “[i]f the Governor decides that the convicted person does not have the mental capacity to understand the nature of the death penalty and why it was imposed on him, he shall have him committed to a Department of Corrections mental health treatment facility.” §922.07(3) (Supp. 1986) (emphasis added). Our recent cases in this area of the law may not be wholly consistent with one another. See Olim v. Wakinekona, 461 U. S. 238 (1983); Hewitt v. Helms, 459 U. S. 460 (1983); Vitek v. Jones, 445 U. S. 480 (1980); Greenholtz v. Nebraska Penal Inmates, 442 U. S. 1 (1979); Meachum v. Fano, 427 U. S. 215 (1976). I do not think this state of the law requires the conclusion that Florida has granted petitioner the sort of entitlement that gives rise to the procedural protections for which he contends.

In any event, I see no reason to reject the Solesbee Court’s conclusion that wholly executive procedures can satisfy due process in the context of a post-trial, postappeal, post-collateral-attack challenge to a State’s effort to carry out *435a lawfully imposed sentence. Creating a constitutional right to a judicial determination of sanity before that sentence may be carried out, whether through the Eighth Amendment or the Due Process Clause, needlessly complicates and postpones still further any finality in this area of the law. The defendant has already had a full trial on the issue of guilt, and a trial on the issue of penalty; the requirement of still a third adjudication offers an invitation to those who have nothing to lose by accepting it to advance entirely spurious claims of insanity. A claim of insanity may be made at any time before sentence and, once rejected, may be raised again; a prisoner found sane two days before execution might claim to have lost his sanity the next day, thus necessitating another judicial determination of his sanity and presumably another stay of his execution. See Nobles v. Georgia, 168 U. S. 398, 405-406 (1897).

Since no State sanctions execution of the insane, the real battle being fought in this case is over what procedures must accompany the inquiry into sanity. The Court reaches the result it does by examining the common law, creating a constitutional right that no State seeks to violate, and then concluding that the common-law procedures are inadequate to protect the newly created but common-law based right. I find it unnecessary to “constitutionalize” the already uniform view that the insane should not be executed, and inappropriate to “selectively incorporate” the common-law practice. I therefore dissent.

10.4 Godinez v. Moran 10.4 Godinez v. Moran

GODINEZ, WARDEN v. MORAN

No. 92-725.

Argued April 21, 1993

Decided June 24, 1993

*390Thomas, J., delivered the opinion of the Court, in which Rehnquist, C. J., and White, O’Connor, and Souter, JJ., joined, and in Parts I, Il-B, and III of which Scalia and Kennedy, JJ., joined. Kennedy, J., filed an opinion concurring in part and concurring in the judgment, in which Scalia, J., joined, post, p. 402. Blackmun, J., filed a dissenting opinion, in which Stevens, J., joined, post, p. 409.

David F. Sarnowski, Chief Deputy Attorney General of Nevada, argued the cause for petitioner. With him on the brief were Frankie Sue Del Papa, Attorney General, and Brooke A. Nielsen, Assistant Attorney General.

Amy L. Wax argued the cause for the United States as amicus curiae urging reversal. With her on the brief were Acting Solicitor General Bryson, Acting Assistant Attorney General Keeney, and Joel M. Gershowitz.

*391Cal J. Potter III, by appointment of the Court, 506 U. S. 1046, argued the cause for respondent. With him on the brief was Edward M. Chikofsky*

Justice Thomas

delivered the opinion of the Court.

This case presents the question whether the competency standard for pleading guilty or waiving the right to counsel is higher than the competency standard for standing trial. We hold that it is not.

I

On August 2, 1984, in the early hours of the morning, respondent entered the Red Pearl Saloon in Las Vegas, Nevada, and shot the bartender and a patron four times each with an automatic pistol. He then walked behind the bar and removed the cash register. Nine days later, respondent arrived at the apartment of his former wife and opened fire on her; five of his seven shots hit their target. Respondent then shot himself in the abdomen and attempted, without success, to slit his wrists. Of the four victims of respondent’s gunshots, only respondent himself survived. On August 13, respondent summoned police to his hospital bed and confessed to the killings.

After respondent pleaded not guilty to three counts of first-degree murder, the trial court ordered that he be examined by a pair of psychiatrists, both of whom concluded that he was competent to stand trial.1 The State thereafter an*392nounced its intention to seek the death penalty. On November 28, 1984, 2lk months after the psychiatric evaluations, respondent again appeared before the trial court. At this time respondent informed the court that he wished to discharge his attorneys and change his pleas to guilty. The reason for the request, according to respondent, was to prevent the presentation of mitigating evidence at his sentencing.

On the basis of the psychiatric reports, the trial court found that respondent

“is competent in that he knew the nature and quality of his acts, had the capacity to determine right from wrong; that he understands the nature of the criminal charges against him and is able to assist in his defense of such charges, or against the pronouncement of the judgment thereafter; that he knows the consequences of entering a plea of guilty to the charges; and that he can intelligently and knowingly waive his constitutional right to assistance of an attorney.” App. 21.

The court advised respondent that he had a right both to the assistance of counsel and to self-representation, warned him of the “dangers and disadvantages” of self-representation, id., at 22, inquired into his understanding of the proceedings and his awareness of his rights, and asked why he had chosen to represent himself. It then accepted respondent’s waiver of counsel. The court also accepted respondent’s guilty pleas, but not before it had determined that respondent was not pleading guilty in response to threats or promises, that he understood the nature of the charges against him and the consequences of pleading guilty, that he was aware of the *393rights he was giving up, and that there was a factual basis for the pleas. The trial court explicitly found that respondent was “knowingly and intelligently” waiving his right to the assistance of counsel, ibid,., and that his guilty pleas were “freely and voluntarily” given, id., at 64.2

On January 21, 1985, a three-judge court sentenced respondent to death for each of the murders. The Supreme Court of Nevada affirmed respondent’s sentences for the Red Pearl Saloon murders, but reversed his sentence for the murder of his ex-wife and remanded for imposition of a life sentence without the possibility of parole. Moran v. State, 103 Nev. 138, 734 P. 2d 712 (1987).

On July 30, 1987, respondent filed a petition for post-conviction relief in state court. Following an evidentiary hearing, the trial court rejected respondent’s claim that he was “mentally incompetent to represent himself,” concluding that “the record clearly shows that he was examined by two psychiatrists both of whom declared [him] competent.” App. to Pet. for Cert. D-8. The Supreme Court of Nevada dismissed respondent’s appeal, Moran v. Warden, 105 Nev. 1041, 810 P. 2d 335, and we denied certiorari, 493 U. S. 874 (1989).

Respondent then filed a habeas petition in the United States District Court for the District of Nevada. The District Court denied the petition, but the Ninth Circuit reversed. 972 F. 2d 263 (1992). The Court of Appeals concluded that the “record in this case” should have led the trial court to “entertai[n] a good faith doubt about [respondent’s] competency to make a voluntary, knowing, and intelligent *394waiver of constitutional rights,” id., at 265,3 and that the Due Process Clause therefore “required the court to hold a hearing to evaluate and determine [respondent’s] competency ... before it accepted his decision to discharge counsel and change his pleas,” ibid. Rejecting petitioner’s argument that the trial court’s error was “cured by the postconviction hearing,” ibid., and that the competency determination that followed the hearing was entitled to deference under 28 U. S. C. § 2254(d), the Court of Appeals held that “the state court’s postconviction ruling was premised on the wrong legal standard of competency,” 972 F. 2d, at 266. “Competency to waive constitutional rights,” according to the Court of Appeals, “requires a higher level of mental functioning than that required to stand trial”; while a defendant is competent to stand trial if he has “a rational and factual understanding of the proceedings and is capable of assisting his counsel,” a defendant is competent to waive counsel or plead guilty only if he has “the capacity for ‘reasoned choice’ among the alternatives available to him.” Ibid. The Court of Appeals determined that the trial court had “erroneously applied the standard for evaluating competency to stand trial, instead of the correct ‘reasoned choice’ standard,” id., at 266-267, and further concluded that when examined “in light of the correct legal standard,” the record did not support a finding that respondent was “mentally capable of the reasoned choice required for a valid waiver of constitutional rights,” id., at 267.4 The Court of Appeals accordingly in*395structed the District Court to issue the writ of habeas corpus within 60 days, “unless the state court allows [respondent] to withdraw his guilty pleas, enter new pleas, and proceed to trial with the assistance of counsel.” Id., at 268.

Whether the competency standard for pleading guilty or waiving the right to counsel is higher than the competency standard for standing trial is a question that has divided the Federal Courts of Appeals5 and state courts of last re*396sort.6 We granted certiorari to resolve the conflict. 506 U. S. 1033 (1992).

II

A criminal defendant may not be tried unless he is competent, Pate v. Robinson, 383 U. S. 375, 378 (1966), and he may not waive his right to counsel or plead guilty unless he does so “competently and intelligently,” Johnson v. Zerbst, 304 U. S. 458, 468 (1938); accord, Brady v. United States, 397 U. S. 742, 758 (1970). In Dusky v. United States, 362 U. S. 402 (1960) (per curiam), we held that the standard for competence to stand trial is whether the defendant has “sufficient present ability to consult with his lawyer with a reasonable degree of rational understanding” and has “a rational as well as factual understanding of the proceedings against him.” Ibid, (internal quotation marks omitted). Accord, Drope v. Missouri, 420 U. S. 162, 171 (1975) (“[A] person whose mental condition is such that he lacks the capacity to understand the nature and object of the proceedings against him, to consult with counsel, and to assist in preparing his defense may not be subjected to a trial”). While we have described the standard for competence to stand trial, however, we have never expressly articulated a standard for competence to plead guilty or to waive the right to the assistance of counsel.

Relying in large part upon our decision in Westbrook v. Arizona, 384 U. S. 150 (1966) (per curiam), the Ninth Circuit adheres to the view that the competency standard for pleading guilty or waiving the right to counsel is higher than the competency standard for standing trial. See Sieling v. Eyman, 478 F. 2d 211, 214-215 (1973) (first Ninth Circuit *397decision applying heightened standard). In Westbrook, a two-paragraph per curiam opinion, we vacated the lower court’s judgment affirming the petitioner’s conviction, because there had been “a hearing on the issue of [the petitioner’s] competence to stand trial,” but “no hearing or inquiry into the issue of his competence to waive his constitutional right to the assistance of counsel.” 384 U. S., at 150. The Ninth Circuit has reasoned that the “clear implication” of Westbrook is that the Dusky formulation is not “a high enough standard” for determining whether a defendant is competent to waive a constitutional right. Sieling, supra, at 214.7 We think the Ninth Circuit has read too much into Westbrook, and we think it errs in applying two different competency standards.8

A

The standard adopted by the Ninth Circuit is whether a defendant who seeks to plead guilty or waive counsel has the capacity for “reasoned choice” among the alternatives available to him. How this standard is different from (much less higher than) the Dusky standard — whether the defendant has a “ratioial understanding” of the proceedings — is not readily apparent to us. In fact, respondent himself opposed certiorari on the ground that the difference between the two standards is merely one of “terminology,” Brief in Opposition 4, and he devotes little space in his brief on the merits to a defense of the Ninth Circuit’s standard, see, e. g., Brief for *398Respondent 17-18, 27, 32; see also Tr. of Oral Arg. 33 (“Due process does not require [a] higher standard, [it] requires a separate inquiry”).9 But even assuming that there is some meaningful distinction between the capacity for “reasoned choice” and a “rational understanding” of the proceedings, we reject the notion that competence to plead guilty or to waive the right to counsel must be measured by a standard that is higher than (or even different from) the Dusky standard.

We begin with the guilty plea. A defendant who stands trial is likely to be presented with choices that entail relinquishment of the same rights that are relinquished by a defendant who pleads guilty: He will ordinarily have to decide whether to waive his “privilege against compulsory self-incrimination,” Boykin v. Alabama, 395 U. S. 238, 243 (1969), by taking the witness stand; if the option is available, he may have to decide whether to waive his “right to trial by jury,” ibid,.; and, in consultation with counsel, he may have to decide whether to waive his “right to confront [his] accusers,” ibid., by declining to cross-examine witnesses for the prosecution. A defendant who pleads not guilty, moreover, faces still other strategic choices: In consultation with his attorney, he may be called upon to decide, among other things, whether (and how) to put on a defense and whether to raise one or more affirmative defenses. In sum, all criminal defendants — not merely those who plead guilty — may be required to make important decisions once criminal proceedings have been initiated. And while the decision to plead guilty is undeniably a profound one, it is no more complicated than the sum total of decisions that a defendant may be called upon to make during the course of a trial. (The decision to plead guilty is also made over a shorter period of *399time, without the distraction and burden of a trial.) This being so, we can conceive of no basis for demanding a higher level of competence for those defendants who choose to plead guilty. If the Dusky standard is adequate for defendants who plead not guilty, it is necessarily adequate for those who plead guilty.

Nor do we think that a defendant who waives his right to the assistance of counsel must be more competent than a defendant who does not, since there is no reason to believe that the decision to waive counsel requires an appreciably higher level of mental functioning than the decision to waive other constitutional rights. Respondent suggests that a higher competency standard is necessary because a defendant who represents himself “ ‘must have greater powers of comprehension, judgment, and reason than would be necessary to stand trial with the aid of an attorney.’ ” Brief for Respondent 26 (quoting Silten & Tullís, Mental Competency in Criminal Proceedings, 28 Hastings L. J. 1053, 1068 (1977)). Accord, Brief for National Association of Criminal Defense Lawyers as Amicus Curiae 10-12. But this argument has a flawed premise; the competence that is required of a defendant seeking to waive his right to counsel is the competence to waive the right, not the competence to represent himself.10 In Faretta v. California, 422 U. S. 806 (1975), we *400held that a defendant choosing self-representation must do so “competently and intelligently/’ id., at 835, but we made it clear that the defendant’s “technical legal knowledge” is “not relevant” to the determination whether he is competent to waive his right to counsel, id., at 836, and we emphasized that although the defendant “may conduct his own defense ultimately to his own detriment, his choice must be honored,” id., at 834. Thus, while “[i]t is undeniable that in most criminal prosecutions defendants could better defend with counsel’s guidance than by their own unskilled efforts,” ibid., a criminal defendant’s ability to represent himself has no bearing upon his competence to choose self-representation.11

B

A finding that a defendant is competent to stand trial, however, is not all that is necessary before he may be permitted to plead guilty or waive his right to counsel. In addition to determining that a defendant who seeks to plead guilty or waive counsel is competent, a trial court must satisfy itself that the waiver of his constitutional rights is knowing and voluntary. Parke v. Raley, 506 U. S. 20, 28-29 (1992) (guilty plea); Faretta, supra, at 835 (waiver of counsel). In this *401sense there is a “heightened” standard for pleading guilty and for waiving the right to counsel, but it is not a heightened standard of competence.12

This two-part inquiry13 is what we had in mind in West-brook. When we distinguished between “competence to stand trial” and “competence to waive [the] constitutional right to the assistance of counsel,” 384 U. S., at 150, we were using “competence to waive” as a shorthand for the “intelligent and competent waiver” requirement of Johnson v. Zerbst. This much is clear from the fact that we quoted that very language from Zerbst immediately after noting that the trial court had not determined whether the petitioner was competent to waive his right to counsel. See 384 U. S., at 150 (“ ‘This protecting duty imposes the serious and weighty responsibility upon the trial judge of determining whether there is an intelligent and competent waiver by the accused’ ”) (quoting Johnson v. Zerbst, 304 U. S., at 465). Thus, Westbrook stands only for the unremarkable proposi*402tion that when a defendant seeks to waive his right to counsel, a determination that he is competent to stand trial is not enough; the waiver must also be intelligent and voluntary-before it can be accepted.14

Ill

Requiring that a criminal defendant be competent has a modest aim: It seeks to ensure that he has the capacity to understand the proceedings and to assist counsel. While psychiatrists and scholars may find it useful to classify the various kinds and degrees of competence, and while States are free to adopt competency standards that are more elaborate than the Dusky formulation, the Due Process Clause does not impose these additional requirements. Cf. Medina v. California, 505 U. S. 437, 446-453 (1992). The judgment of the Court of Appeals is reversed, and the case is remanded for further proceedings consistent with this opinion.

So ordered.

Justice Kennedy,

with whom

Justice Scalia joins, concurring in part and concurring in the judgment.

I am in full agreement with the Court’s decision that the competency standard for pleading guilty and waiving the right to counsel is the same as the test of competency to stand trial. As I have some reservations about one part of the Court’s opinion and take a somewhat different path to reach my conclusion, it is appropriate to make some further observations.

The Court compares the types of decisions made by one who goes to trial with the decisions required to plead guilty and waive the right to counsel. This comparison seems to suggest that there may have been a heightened standard of *403competency required by the Due Process Clause if the decisions were not equivalent. I have serious doubts about that proposition. In discussing the standard for a criminal defendant’s competency to make decisions affecting his case, we should not confuse the content of the standard with the occasions for its application.

We must leave aside in this case any question whether a defendant is absolved of criminal responsibility due to his mental state at the time he committed criminal acts and any later question about whether the defendant has the minimum competence necessary to undergo his sentence. What is at issue here is whether the defendant has sufficient competence to take part in a criminal proceeding and to make the decisions throughout its course. This is not to imply that mental competence is the only aspect of a defendant’s state of mind that is relevant during criminal proceedings. Whether the defendant has made a knowing, intelligent, and voluntary decision to make certain fundamental choices during the course of criminal proceedings is another subject of judicial inquiry. That both questions might be implicated at any given point, however, does not mean that the inquiries cease to be discrete. And as it comes to us, this case involves only the standard for determining competency.

This Court set forth the standard for competency to stand trial in Dusky v. United States, 362 U. S. 402 (1960) (per curiam): “[T]he ‘test must be whether [the defendant] has sufficient present ability to consult with his lawyer with a reasonable degree of rational understanding — and whether he has a rational as well as factual understanding of the proceedings against him.’” Ibid. In my view, both the Court of Appeals and respondent read “competency to stand trial” in too narrow a fashion. We have not suggested that the Dusky competency standard applies during the course of, but not before, trial. Instead, that standard is applicable from the time of arraignment through the return of a verdict. Although the Dusky standard refers to “ability to consult *404with [a] lawyer,” the crucial component of the inquiry is the defendant’s possession of “a reasonable degree of rational understanding.” In other words, the focus of the Dusky formulation is on a particular level of mental functioning, which the ability to consult counsel helps identify. The possibility that consultation will occur is not required for the standard to serve its purpose. If a defendant elects to stand trial and to take the foolish course of acting as his own counsel, the law does not for that reason require any added degree of competence. See ante, at 399-400, n. 10.

The Due Process Clause does not mandate different standards of competency at various stages of or for different decisions made during the criminal proceedings. That was never the rule at common law, and it would take some extraordinary showing of the inadequacy of a single standard of competency for us to require States to employ heightened standards. See Medina v. California, 505 U. S. 437, 446-447 (1992). Indeed, we should only overturn Nevada’s use of a single standard if it “ ‘offends some principle of justice so rooted in the traditions and conscience of our people as to be ranked as fundamental.’” Ibid, (quoting Patterson v. New York, 432 U. S. 197, 202 (1977)).

The historical treatment of competency that supports Nevada’s single standard has its roots in English common law. Writing in the 18th century, Blackstone described the effect of a defendant’s incompetence on criminal proceedings:

“[I]f a man in his sound memory commits a capital of-fence, and before arraignment for it, he becomes mad, he ought not to be arraigned for it; because he is not able to plead to it with that advice and caution that he ought. And if, after he has pleaded, the prisoner becomes mad, he shall not be tried; for how can he make his defence?” 4 W. Blackstone, Commentaries *24.

Accord, 1 M. Hale, Pleas of the Crown *34-*35. *405Blackstone drew no distinction between madness for purposes of pleading and madness for purposes of going to trial. An English case arising in the Crown Court in 1865 indicates that a single standard was applied to assess competency at the time of arraignment, the time of pleading, and throughout the course of trial. See Regina v. Southey, 4 Fos. & Fin. 864, 872, n. a, 176 Eng. Rep. 825, 828, n. a (N. P. 1865) (“Assuming the prisoner to be insane at the time of arraignment, he cannot be tried at all, with or without counsel, for, even assuming that he has appointed counsel at a time when he was sane, it is not fit that he should be tried, as he cannot understand the evidence, nor the proceedings, and so is unable to instruct counsel, or to withdraw his authority if he acts improperly, as a prisoner may always do”); id., at 877, n. a, 176 Eng. Rep., at 831, n. a (“[I]f [the defendant] be so insane as not to understand the nature of the proceedings, he cannot plead”).

A number of 19th-century American cases also referred to insanity in a manner that suggested there was a single standard by which competency was to be assessed throughout legal proceedings. See, e. g., Underwood v. People, 32 Mich. 1, 3 (1875) (“[I]nsanity, when discovered, was held at common law to bar any further steps against a prisoner, at whatever stage of the proceedings”); Crocker v. State, 60 Wis. 553, 556, 19 N. W. 435, 436 (1884) (“At common law, if a person, after committing a crime, became insane, he was not arraigned during his insanity, but was remitted to prison until such incapacity was removed. The same was true where he became insane after his plea of not guilty and before trial”); State v. Reed, 41 La. Ann. 581, 582, 7 So. 132 (1889) (“It is elementary that a man cannot plead, or be tried, or convicted, or sentenced, while in a state of insanity”). See also 2 J. Bishop, Commentaries on Law of Criminal Procedure §§ 664, 667 (2d ed. 1872) (“[A] prisoner cannot be tried, sentenced, or punished” unless he is “mentally competent to make a rational defense”).

*406Other American cases describe the standard by which competency is to be measured in a way that supports the idea that a single standard, parallel to that articulated in Dusky, is applied no matter at what point during legal proceedings a competency question should arise. For example, in Freeman v. People, 4 Denio 2 (N. Y. 1847), it was held: “If ... a person arraigned for a crime, is capable of understanding the nature and object of the proceedings going on against him; if he rightly comprehends his own condition in reference to such proceedings, and can conduct his defence in a rational manner, he is, for the purpose of being tried, to be deemed sane.” Id., at 24-25. Because the competency question was posed in Freeman at the time the defendant was to be arraigned, id., at 19, the Freeman court’s conception of competency to stand trial was that of a single standard to be applied throughout.

An even more explicit recitation of this common-law principle is found in Hunt v. State, 27 So. 2d 186 (Ala. 1946). In the course of the opinion in that case, thebe was a discussion of the common-law rule regarding a defendant’s competency to take part in legal proceedings:

“The rule at common law ... is that if at any time while criminal proceedings are pending against a person accused of a crime, the trial court either from observation or upon suggestion of counsel has facts brought to his attention which raise a doubt of the sanity of defendant, the question should be settled before further steps are taken. . . . The broad question to be determined then is whether the defendant is capable of understanding the proceedings and of making his defense, and whether he may have a full, fair and impartial trial.” Id., at 191 (citation omitted).

At common law, therefore, no attempt was made to apply different competency standards to different stages of criminal proceedings or to the variety of decisions that a defend*407ant must make during the course of those proceedings. See Commonwealth v. Woelfel, 88 S. W. 1061, 1062 (Ky. 1905); Jordan v. State, 135 S. W. 327, 328-329 (Tenn. 1911); State v. Seminary, 115 So. 370, 371-372 (La. 1927); State ex rel. Townsend v. Bushong, 146 Ohio St. 271, 272, 65 N. E. 2d 407, 408 (1946) (per curiam); Moss v. Hunter, 167 F. 2d 683, 684-685 (CA10 1948). Commentators have agreed that the common-law standard of competency to stand trial, which parallels the Dusky standard, has been applied throughout criminal proceedings, not just to the formal trial. See H. Weihofen, Mental Disorder as a Criminal Defense 428-429, 431 (1954) (“It has long been the rule of the common law that a person cannot be required to plead to an indictment or be tried for a crime while he is so mentally disordered as to be incapable of making a rational defense”); S. Brakel, J. Parry, and A. Weiner, The Mentally Disabled and the Law 695-696 (3d ed. 1985) (“It has traditionally been presumed that competency to stand trial means competency to participate in all phases of the trial process, including such pretrial activities as deciding how to plead, participating in plea bargaining, and deciding whether to assert or waive the right to counsel”).

That the common law did not adopt heightened competency standards is readily understood when one considers the difficulties that would be associated with more than one standard. The standard applicable at a given point in a trial could be difficult to ascertain. For instance, if a defendant decides to change his plea to guilty after a trial has commenced, one court might apply the competency standard for undergoing trial while another court might use the standard for pleading guilty. In addition, the subtle nuances among different standards are likely to be difficult to differentiate, as evidenced by the lack of any clear distinction between a “rational understanding” and a “reasoned choice” in this case. See ante, at 398.

*408It is true, of course, that if a defendant stands trial instead of pleading guilty, there will be more occasions for the trial court to observe the condition of the defendant to determine his mental competence. Trial courts have the obligation of conducting a hearing whenever there is sufficient doubt concerning a defendant’s competence. See Drope v. Missouri, 420 U. S. 162, 180-181 (1975). The standard by which competency is assessed, however, does not change. Respondent’s counsel conceded as much during oral argument, making no attempt to defend the contrary position of the Court of Appeals. See, e. g., Tr. of Oral Arg. 22 (“This is not a case of heightened standards”); id., at 31 (“We didn’t argue a heightened standard. We did not argue a heightened standard to the Ninth Circuit, nor did we necessarily argue a heightened standard at any juncture in this case”); id., at 33 (“Due process does not require this higher standard, but requires a separate inquiry”).

A single standard of competency to be applied throughout criminal proceedings does not offend any “ ‘principle of justice so rooted in the traditions and conscience of our people as to be ranked as fundamental.’ ” Medina, 505 U. S., at 446. Nothing in our case law compels a contrary conclusion, and adoption of a rule setting out varying competency standards for each decision and stage of a criminal proceeding would disrupt the orderly course of trial and, from the standpoint of all parties, prove unworkable both at trial and on appellate review.

I would avoid the difficult comparisons engaged in by the Court. In my view, due process does not preclude Nevada’s use of a single competency standard for all aspects of the criminal proceeding. Respondent’s decision to plead guilty and his decision to waive counsel were grave choices for him to make, but as the Court demonstrates in Part II-B, there is a heightened standard, albeit not one concerned with competence, that must be met before a defendant is allowed to make those decisions.

*409With these observations, I concur in the judgment and in Parts I, II-B, and III of the Court’s opinion.

Justice Blackmun,

with whom

Justice Stevens joins, dissenting.

Today, the majority holds that a standard of competence designed to measure a defendant’s ability to consult with counsel and to assist in preparing his defense is constitutionally adequate to assess a defendant’s competence to waive the right to counsel and represent himself. In so doing, the majority upholds the death sentence for a person whose decision to discharge counsel, plead guilty, and present no defense well may have been the product of medication or mental illness. I believe the majority’s analysis is contrary to both common sense and longstanding case law. Therefore, I dissent.

I

As a preliminary matter, the circumstances under which respondent Richard Allan Moran waived his right to an attorney and pleaded guilty to capital murder bear elaboration. For, although the majority’s exposition of the events is accurate, the most significant facts are omitted or relegated to footnotes.

In August 1984, after killing three people and wounding himself in an attempt to commit suicide, Moran was charged in a Nevada state court with three counts of capital murder. He pleaded not guilty to all charges, and the trial court ordered a psychiatric evaluation. At this stage, Moran’s competence to represent himself was not at issue.

The two psychiatrists who examined him therefore focused solely upon his capacity to stand trial with the assistance of counsel. Dr. Jack A. Jurasky found Moran to be “in full control of his faculties insofar as his ability to aid counsel, assist in his own defense, recall evidence and to give testimony if called upon to do so.” App. 8. Dr. Jurasky, however, did express some reservations, observing: “Psy*410chologically, and perhaps legally speaking, this man, because he is expressing and feeling considerable remorse and guilt, may be inclined to exert less effort towards his own defense.” Ibid. Nevertheless, under the circumstances, Dr. Jurasky felt that Moran’s depressed state of mind was not “necessarily a major consideration.” Ibid. Dr. William D. O’Gorman also characterized Moran as “very depressed,” remarking that he “showed much tearing in talking about the episodes that led up to his present incarceration, particularly in talking about his ex-wife.” Id., at 15-16. But Dr. O’Gorman ultimately concluded that Moran “is knowledgeable of the charges being made against him” and “can assist his attorney, in his own defense, if he so desires.” Id., at 17.

In November 1984, just three months after his suicide attempt, Moran appeared in court seeking to discharge his public defender, waive his right to counsel, and plead guilty to all three charges of capital murder. When asked to explain the dramatic change in his chosen course of action, Moran responded that he wished to represent himself because he opposed all efforts to mount a defense. His purpose, specifically, was to prevent the presentation of any mitigating evidence on his behalf at the sentencing phase of the proceeding. The trial judge inquired whether Moran was “presently under the influence of any drug or alcohol,” and Moran replied: “Just what they give me in, you know, medications.” Id., at 33. Despite Moran’s affirmative answer, the trial judge failed to question him further regarding the type, dosage, or effect of the “medications” to which he referred. Had the trial judge done so, he would have discovered that Moran was being administered simultaneously four different prescription drugs — phenobarbital, dilantin, inderal, and vistaril. Moran later testified to the numbing effect of these drugs, stating: “I guess I really didn’t care about anything .... I wasn’t very concerned about anything that *411was going on ... as far as the proceedings and everything were going.” Id., at 92.1

Disregarding the mounting evidence of Moran’s disturbed mental state, the trial judge accepted Moran’s waiver of counsel and guilty pleas after posing a series of routine questions regarding his understanding of his legal rights and the offenses, to which Moran gave largely monosyllabic answers. In a string of affirmative responses, Moran purported to acknowledge that he knew the import of waiving his constitutional rights, that he understood the charges against him, and that he was, in fact, guilty of those charges. One part of this exchange, however, highlights the mechanical character of Moran’s answers to the questions. When the trial judge asked him whether he killed his ex-wife “deliberately, with premeditation and malice aforethought,” Moran unexpectedly responded: “No. I didn’t do it — I mean, I wasn’t looking to kill her, but she ended up dead.” Id., at 58. Instead of probing further, the trial judge simply repeated the question, inquiring again whether Moran had acted deliberately. Once again, Moran replied: “I don’t know. I mean, I don’t know what you mean by deliberately. I mean, I pulled the trigger on purpose, but I didn’t plan on doing it; you know what I mean?” Id., at 59. Ignoring the ambiguity of Moran’s responses, the trial judge reframed the question to elicit an affirmative answer, stating: “Well, I’ve previously explained to you what is meant by deliberation and premeditation. Deliberate means that you arrived at or determined as a result of careful thought and weighing the consideration *412for and against the proposed action. Did you do that?” This time, Moran responded: “Yes.” Ibid.

It was only after prodding Moran through the plea colloquy in this manner that the trial judge concluded that he was competent to stand trial and that he voluntarily and intelligently had waived his right to counsel. Accordingly, Moran was allowed to plead guilty and appear without counsel at his sentencing hearing. Moran presented no defense, called no witness, and offered no mitigating evidence on his own behalf. Not surprisingly, he was sentenced to death.

II

It is axiomatic by now that criminal prosecution of an incompetent defendant offends the Due Process Clause of the Fourteenth Amendment. See Medina v. California, 505 U. S. 437 (1992); Riggins v. Nevada, 504 U. S. 127, 138 (1992) (Kennedy, J., concurring); Drope v. Missouri, 420 U. S. 162, 171 (1975); Pate v. Robinson, 383 U. S. 375, 378 (1966). The majority does not deny this principle, nor does it dispute the standard that has been set for competence to stand trial with the assistance of counsel: whether the accused possesses “the capacity to understand the nature and object of the proceedings against him, to consult with counsel, and to assist in preparing his defense.” Drope, 420 U. S., at 171. Accord, Dusky v. United States, 362 U. S. 402 (1960). My disagreement with the majority turns, then, upon another standard— the one for assessing a defendant’s competence to waive counsel and represent himself.

The majority “reject[s] the notion that competence to plead guilty or to waive the right to counsel must be measured by a standard that is higher than (or even different from)” the standard for competence to stand trial articulated in Dusky and Drope. Ante, at 398. But the standard for competence to stand trial is specifically designed to measure a defendant’s ability to “consult with counsel” and to “assist in preparing his defense.” A finding that a defendant is *413competent to stand trial establishes only that he is capable of aiding his attorney in making the critical decisions required at trial or in plea negotiations. The reliability or even relevance of such a finding vanishes when its basic premise — that counsel will be present — ceases to exist. The question is no longer whether the defendant can proceed with an attorney, but whether he can proceed alone and uncounseled. I do not believe we place an excessive burden upon a trial court by requiring it to conduct a specific inquiry into that question at the juncture when a defendant whose competency already has been questioned seeks to waive counsel and represent himself.

The majority concludes that there is no need for such a hearing because a defendant who is found competent to stand trial with the assistance of counsel is, ipso facto, competent to discharge counsel and represent himself. But the majority cannot isolate the term “competent” and apply it in a vacuum, divorced from its specific context. A person who is “competent” to play basketball is not thereby “competent” to play the violin. The majority’s monolithic approach to competency is true to neither life nor the law. Competency for one purpose does not necessarily translate to competency for another purpose. See Bonnie, The Competence of Criminal Defendants: A Theoretical Reformulation, 10 Behav. Sci. & L. 291, 299 (1992); R. Roesch & S. Golding, Competency to Stand Trial 10-13 (1980). Consistent with this commonsense notion, our cases always have recognized that “a defendant’s mental condition may be relevant to more than one legal issue, each governed by distinct rules reflecting quite different policies.” Drope, 420 U. S., at 176. See Jackson v. Indiana, 406 U. S. 715, 739 (1972). To this end, this Court has required competency evaluations to be specifically tailored to the context and purpose of a proceeding. See Rees v. Peyton, 384 U. S. 312, 314 (1966) (directing court “to determine [petitioner’s] mental competence in the present posture of things”).

*414In Massey v. Moore, 348 U. S. 105, 108 (1954), for example, the Court ruled that a defendant who had been found competent to stand trial with the assistance of counsel should have been given a hearing as to his competency to represent himself because “[o]ne might not be insane in the sense of being incapable of standing trial and yet lack the capacity to stand trial without benefit of counsel.”2 And in Westbrook v. Arizona, 384 U. S. 150 (1966), the Court reiterated the requirement that the determination of a defendant’s competency be tailored to the particular capacity in question, observing: “Although petitioner received a hearing on the issue of his competence to stand trial, there appears to have been no hearing or inquiry into the issue of his competence to waive his constitutional right to the assistance of counsel and proceed, as he did, to conduct his own defense.” See also Medina, 505 U. S., at 446-448 (distinguishing between a claim of incompetence and a plea of not guilty by reason of insanity); Riggins, 504 U. S., at 140-144 (Kennedy, J., concurring) (distinguishing between functional competence and competence to stand trial).

Although the Court never has articulated explicitly the standard for determining competency to represent oneself, it has hinted at its contours. In Rees v. Peyton, supra, it required an evaluation of competence that was designed to measure the abilities necessary for a defendant to make a decision under analogous circumstances. In that case, a capital defendant who had filed a petition for certiorari ordered his attorney to withdraw the petition and forgo further legal proceedings. The petitioner’s counsel advised the Court that he could not conscientiously do so without a psychiatric examination of his client because there was some doubt as to *415his client’s mental competency. Under those circumstances, this Court directed the lower court to conduct an inquiry as to whether the defendant possessed the “capacity to appreciate his position and make a rational choice with respect to continuing or abandoning further litigation or on the other hand whether he is suffering from a mental disease, disorder, or defect which may substantially affect his capacity in the premises.” 384 U. S., at 314 (emphasis added). Certainly the competency required for a capital defendant to proceed without the advice of counsel at trial or in plea negotiations should be no less than the competency required for a capital defendant to proceed against the advice of counsel to . withdraw a petition for certiorari. The standard applied by the Ninth Circuit in this case — the “reasoned choice” standard— closely approximates the “rational choice” standard set forth in Rees3

Disregarding the plain language of Westbrook and Massey, the majority in effect overrules those cases sub silentio4 From the constitutional right of self-representation established in Faretta v. California, 422 U. S. 806 (1975), the majority extrapolates that “a criminal defendant’s ability to represent himself has no bearing upon his competence to choose *416self-representation.” Ante, at 400. But Faretta does not confer upon an incompetent defendant a constitutional right to conduct his own defense. Indeed, Faretta himself was “literate, competent, and understanding,” and the record showed that “he was voluntarily exercising his informed free will.” 422 U. S., at 835. “Although a defendant need not himself have the skill and experience of a lawyer,” Faretta’s right of self-representation is confined to those who are able to choose it “competently and intelligently.” Ibid. The Faretta Court was careful to emphasize that the record must establish that the defendant “ ‘knows what he is doing and his choice is made with eyes open.’” Ibid., quoting Adams v. United States ex rel. McCann, 317 U. S. 269, 279 (1942).

The majority asserts that “the competence that is required of a defendant seeking to waive his right to counsel is the competence to waive the right, not the competence to represent himself.” Ante, at 399. But this assertion is simply incorrect. The majority’s attempt to extricate the competence to waive the right to counsel from the competence to represent oneself is unavailing, because the former decision necessarily entails the latter. It is obvious that a defendant who waives counsel must represent himself. Even Moran, who pleaded guilty, was required to defend himself during the penalty phase of the proceedings. And a defendant who is utterly incapable of conducting his own defense cannot be considered “competent” to make such a decision, any more than a person who chooses to leap out of a window in the belief that he can fly can be considered “competent” to make such a choice.

The record in this case gives rise to grave doubts regarding respondent Moran’s ability to discharge counsel and represent himself. Just a few months after he attempted to commit suicide, Moran essentially volunteered himself for execution: He sought to waive the right to counsel, to plead guilty to capital murder, and to prevent the presentation of any mitigating evidence on his behalf. The psychiatrists’ re*417ports supplied one explanation for Moran’s self-destructive behavior: his deep depression. And Moran’s own testimony suggested another: the fact that he was being administered simultaneously four different prescription medications. It has been recognized that such drugs often possess side effects that may “compromise the right of a medicated criminal defendant to receive a fair trial... by rendering him unable or unwilling to assist counsel.” Riggins, 504 U. S., at 142 (Kennedy, J., concurring). Moran’s plea colloquy only augments the manifold causes for concern by suggesting that his waivers and his assent to the charges against him were not rendered in a truly voluntary and intelligent fashion. Upon this evidence, there can be no doubt that the trial judge should have conducted another competency evaluation to determine Moran’s capacity to waive the right to counsel and represent himself, instead of relying upon the psychiatrists’ reports that he was able to stand trial with the assistance of counsel.5

To try, convict, and punish one so helpless to defend himself contravenes fundamental principles of fairness and impugns the integrity of our criminal justice system. I cannot condone the decision to accept, without further inquiry, the self-destructive “choice” of a person who was so deeply medicated and who might well have been severely mentally ill. I dissent.